You are on page 1of 43

MKSAP: ENDOCRINOLOGY & METABOLISM

 Treat hyperthyroidism 2/2 grave’s disease with PTU if in first trimester pregnancy.
 Grave’s disease: thyromegaly, thyroid bruit, elevated thyroid-stimulating immunoglobulin index, hyperthyroidism.
 Methimazole SE in first trimester of pregnancy: aplasia cutis (absence of a portion of skin on scalp in localized or
widespread area), and choanal atresia (blockage of posterior nasal passage 2/2 failed recanalization of nasal fossae
during fetal development). Once organogenesis is complete, methimazole can be substituted for PTU.
 Methimazole is preferred in hyperthyroidism because it has longer intrathyroidal half-life and can be given once daily.
 Radioactive iodine is contraindicated when pregnant; it may destroy fetal thyroid.
 MEN2A: multiple endocrine neoplasia: mutation in RET proto-oncogene: pheochromocytoma (usually benign and
intra-adrenal), medullary thyroid cancer, primary hyperparathyroidism 2/2 glandular hyperplasia.
 Hyperparathyroidism: causes hypercalcemia (polydipsia, polyuria, constipation). hyperparathyroidism may be found
incidentally during evaluation for osteoporosis or nephrolithiasis.
 MEN1: usually 1 mutated allele of MEN1 gene is inherited, and a somatic mutation in the other allele is later acquired
which results in formation of neoplasia: insulinoma (pancreatic mass), prolactinoma (pituitary), primary
hyperparathyroidism 2/2 parathyroid adenoma.
 Neurofibromatosis type 1: neurofibroma, café-au-lait spots, pheochromocytoma.
 During pregnancy, prolactinomas can grow 2/2 estrogenic stimulation. Evaluate with formal visual field testing even
if patient without symptoms (bitemporal hemianopsia). Risk of growth depends on the size of the prolactinoma prior
to pregnancy. If microadenoma (<10mm), risk is low, whereas macroadenoma (>10mm), risk is higher. Significant
expansion may cause vision loss 2/2 compression of optic chiasm. Prolactin level is of no utility; pregnancy state and
adenoma both causes hyperprolactinemia. Tx with bromocriptine (dopamine agonist); but unclear safety in pregnancy.
 For hypoglycemia overnight, check blood glucose at 3am. Etiologies: overnight hypoglycemia, dawn phenomenon, or
inadequate insulin doses. Normally, physiologic release of catecholamines, cortisol, and growth hormones in AM are
endogenously produced, which stimulates glucose production from liver. Overnight hypoglycemia may be 2/2
overtreatment of DM, or prolonged effects of recent physical exertion (which leads to low glucose values 2/2
amplified release of catecholamines, cortisol, growth hormone, and glucagon).
 Somogyi effect: rebound hyperglycemia: elevated AM glucose 2/2 release of compensatory hormones 2/2 prolonged
nocturnal hypoglycemia.
 Dawn phenomenon: fasting hyperglycemia in setting of inadequate basal insulin coverage to maintain endogenous
glucose value within normal range.
 Cushing syndrome screening tests: low-dose dexamethasone suppression test-LDST (both standard and overnight),
24-hour urine free cortisol (UFC), or late-night salivary cortisol. +CS diagnosis if UFC and failure to suppress
cortisol levels following an overnight LDST. Next test, check serum ACTH levels.
 CS divided into ACTH dependent (>20), and ACTH independent CS.
 CT abdomen adrenal gland protocol to eval for adenomas/carcinoma of the adrenal cortex (MCC ACTH independent)
 ACTH dependent CS: corticotrophic pituitary adenoma. Eval with inferior petrosal sinus sampling and MRI brain.
 Pituitary adenoma is MCC of CS overall.
 Ketosis prone type 2 DM: older, obese, black/Hispanic. Assess fasting C-peptide and FBG (pancreatic beta cell
function) prior to switching from insulin to oral therapy. Heterogeneous condition; presence of autoantibodies are
variable. Beware that prolonged hyperglycemia in setting of DKA causes toxic effect on pancreatic beta cells; never
measure the fasting C-peptide and FBG when acutely ill. Repeat labs 1-2 weeks after correction of acidosis to better
assess the pancreatic function. If fasting C-peptide >1 or if glucagon-stimulated C-peptide >1.5, the patient’s beta-cell
function is preserved (which means you can appropriately switch from insulin to PO meds). If patients switches from
insulin to metformin/sulfonylureas, and develops worsening hyperglycemia or DKA, you need to restart them on
insulin therapy.
 Pituitary apoplexy: acute pituitary hemorrhage, requires stress dose steroids and urgent transphenoidal decompression
or resection of the hemorrhage to preserve vision. Usually hemorrhage is within preexisting prolactinoma (hx of
headaches, loss of libido, erectile dysfunction). Prolactinomas are usually treated with bromocriptine (dopamine
agonists), but for vision changes, surgical intervention is required to decrease pressure on optic chiasm. Give stress
dose steroids because of secondary cortisol (ACTH) deficiency; no need to assess other pituitary function. In next 2-4
weeks, TFT need to be done, but since T4 has long half-life, doesn’t need emergent management.
 Apoplexy: unconsciousness or incapacity 2/2 cerebral hemorrhage or stroke.
 Thyroid nodule management/workup: US evaluation; US is more predictive of malignancy than size is. Features of
cancer: hypoechogenicity, microcalcifications, irregular margins, increased intranodular Doppler flow. Continue with
FNA to determine if malignant vs benign if the hypoechoic nodule is >1.5cm (American thyroid society guidelines).
CT not as sensitive for thyroid nodules, unless substernal goiter into the mediastinum. Levothyroxine therapy to
suppress growth of benign nodules is not recommended. No significant effect on nodule volume, also side effect of
thyrotoxicosis with cardiovascular complications. Thyroglobulin level is not useful for assessing malignancy. Thyroid
scan with technetium is used for determining functional status of nodules; isotope scanning is useful if nodule is
accompanied with low serum TSH (hyperfunctioning nodule). Nodules that are hyperfunctioning usually don’t need
FNA as the vast majority are benign. Normal TSH is likely to yield an indeterminate (warm/cold) thyroid scan.
 Primary amenorrhea: lack of menses by 16 yo with normal body hair pattern and normal breast development. Rule out
pregnancy first. 50% will have chromosomal abnormality.
 Turner syndrome: primary ovarian failure (early follicular depletion, ovaries devoid of follicles and oocytes), short
stature, loss of a portion, or all of one X chromosome. Turner is MCC of primary amenorrhea. Turner also associated
with higher incidence of cardiovascular disease, metabolic syndrome, and thyroid dysfunction. May have primary or
secondary amenorrhea, and frequently have normal secondary sexual characteristics. Serum evaluation reveals low
estradiol levels (undetectable), and markedly elevated gonadotropin levels, which is consistent with
hypergonadotropic hypogonadism. Diagnose Turner syndrome with karyotype analysis. Patient may also have fragile
X permutation (usually accompanied with cognitive impairment). Treat Turner syndrome with hormone replacement
of estrogen and cyclic progestin (to prevent endometrial hyperplasia, osteoporosis, and other sequelae of
hypoestrogenism).
 Pituitary prolactinoma causes secondary amenorrhea 2/2 inhibition of gonadotropin releasing hormone by high
prolactin levels.
 Hypothyroidism and hyperthyroidism can both cause secondary amenorrhea. Hypothyroidism results in elevated
thyrotropin-releasing hormone, which stimulates prolactin secretion, causing suppression in gonadotropin secretion.
 Hyperthyroidism causes rapid weight loss, which causes functional hypothalamic amenorrhea.
 Subacute granulomatous thyroiditis: de Quervain thyroiditis: treat thyrotoxicosis with metoprolol. Usually with low
radioactive iodine uptake (RAIU) and a painful thyroid upon palpation. Associated with antecedent viral illness,
which destroys the thyroid follicles and triggers release of preformed thyroid hormone into bloodstream, causing
thyrotoxic phase. During this phase, further release of thyroid hormone ceases, resulting in low RAIU. BB blocks
adrenergic effects of high circulating thyroid hormone levels. Supportive care with NSAIDS for pain; may consider
steroids if severe pain. Methimazole and PTU are ineffective because the thyroid has already released the preformed
hormone into bloodstream.
 Hyperprolactinemia is a common side effect of antipsychotics (risperidone). Antipsychotics block dopamine and
decreases inhibition of prolactin release from pituitary.
 DEXA with T score -1 to -2.5 indicates low bone mass. FRAX defines a 10-year fracture risk for these patients.
 Start patient on therapy if risk of major osteoporotic fracture >20%, or risk of hip fracture >3% on FRAX.
 RF for osteoporosis, fractures: sex, fracture history, femoral neck BMD, steroid use, smoking BMI, age, alcohol.
 Raloxifene: SERM (selective estrogen receptor modulator): treatment option for osteoporosis, which has shown to
increase BMD and reduce risk of vertebral (NOT non-vertebral) fractures. Associated with increased risk of
thromboembolic events & vasomotor symptoms. Treat if >20% and >3%. Okay to use in women with low bone mass.
 Bisphosphonates: first line therapy for osteoporosis. Not used routinely in women with low bone mass. Treat if >20/3
 Vitamin D3: cholecalciferol: metabolite of Vitamin D: commonly used to supplement low Vitamin D levels.
 Euthyroid sick syndrome: nonthyroidal illness syndrome. Don’t order TFT in sick patients with critical illness, as it is
likely to be abnormal. During acute illness, T3 is low initially, and TSH and T4 may decline as illness increases in
severity. Pattern of TFT during acute illness is indistinguishable from central hypothyroidism. Perhaps low TSH and
low T4/T3 levels is an adaptive mechanism response to critical illness in order to alter body’s metabolism and help it
cope and aid in recovery from the acute phase of illness.
 MCC of hypothyroidism is Hashimoto thyroiditis: reduced basal temperature, diastolic hypertension, enlarged thyroid
gland, bradycardia, pallor, dry and cold skin, brittle hair, hoarseness, delayed recovery phase of DTR. Low T4 and
low T3, with elevated TSH.
 Vitamin D deficiency, refractory to aggressive repletion (50K units/weekly x6w) warrants more workup:
malabsorption 2/2 celiac disease. Remove gluten from diet, which improves intestinal lining facilitating absorption.
 Low vitamin D level triggers PTH secretion.
 Ergocalciferol (D2): more readily available in 50K U form; shorter half life. Recommended when Vit D level <10.
 Cholecalciferol (D3): used when vitamin D level 20-30 or for maintenance (1K U/daily). Not for aggressive repletion.
 Clinical discretion if Vitamin D level 10-20.
 Subclinical hypothyroidism: mild elevation of TSH, normal free T4. Family hx of hypothyroidism and positive
thyroid peroxidase antibody increases likelihood of progression to overt thyroid failure. May start levothyroxine if
symptomatic.
 Thyroid stimulating immunoglobulins (TSI): associated with Graves.
 Radioactive iodine uptake scan (RAIU): used in hyperthyroidism. Graveselevated RAIU. Thyroiditis or exposure to
exogenous thyroid hormonelow RAIU despite biochemical hyperthyroidism.
 Indications for surgical parathyroidectomy: impaired renal function (eGFR<60, 24 hour urine calcium>400),
nephrolithiasis, nephrocalcinosis, <50yo, Ca >1mg/dL above upper limit of normal, T score < -2.5 at L spine, total
hip, femoral neck, or distal radius. If conservative or declining surgery, may give cinacalcet (Calcimimetic agent) to
stimulate calcium-sensing receptors of the parathyroid glands to inhibit PTH secretion.
 Fracture risk in osteoporosis (FRAX-fracture risk assessment tool): 10 year fracture risk for patients with T score
from -1.0 to -2.5. FRAX >20% for risk of major osteoporotic fracture, or >3% of major hip fracture.
 Treat with bisphosphonates (beware in renal failure).
 Elements of FRAX score: gender, fracture history, femoral neck BMD, steroid use, smoking, BMI, age, alcohol.
 Evaluate thyroid nodule with RAIU (radioactive iodine uptake scan) who presents with toxic nodule, especially if
multiple nodules were seen on US. The scan will also determine if FNA is indicated. Hyperfunctioning or “hot”
nodules do not need FNA, whereas “cold” or “warm” (similar uptake to surrounding non-nodular thyroid tissue) may
require cytology. If patients present with overt thyrotoxicosis, started methimazole to lower the thyroid hormone
levels prior to additional testing is reasonable. Once hormone levels are near normal, the thionamide should be held 5
days prior to the thyroid scan.
 Surgical removal is reasonable for toxic nodules, or multiple toxic nodules.
 Treat prolactinoma with dopamine agonist (bromocriptine, cabergoline). It is the MCC of hyperprolactinemia.
Microadenoma < 10 mm, macroadenoma > 10 mm in diameter. Prolactinomas are most common type of secretory
pituitary adenoma. May cause mas effect on optic chiasm, invasion into cavernous sinus, secondary hypothyroidism,
and GH deficiency. Conservative medical therapy can normalize prolactin levels, reverse hypogonadism, and shrink
tumors by >50% in >90% of patients, as long as visual acuity is not threatened.
 Octreotide is treatment of choice for acromegaly.
 AMS in diabetics 2/2 hypoglycemia on sulfonylurea therapy: in a patient who is dehydrated with poor PO intake, with
nausea/vomiting, and AKI. Sulfonylureas are renally cleared. Glyburide has a longer half-life than other sulfonylurea
and should be avoided in older patients. Dehydration prolongs the glucose-lowering effects of insulin secretagogue.
 Levothyroxine has a long half-life, allowing once dialing dosing. Missing a few doses won’t cause myxedema coma.
 TFT includes: TSH, T3, and fT4. Rarely, T3 is elevated and T4 is normal in hyperthyroidism.
 T3 levels is not indicated in patients with hypothyroidism because the T3 concentration is conserved and may remain
within normal range even in patients with significant hypothyroidism.
 TPO (thyroid peroxidase) antibody titer is helpful if patients has mildly elevated serum TSH associated with
Hashimoto thyroiditis and at future risk of developing permanent hypothyroidism, but is rarely useful.
 Repeating TFT in 6 weeks is appropriate if the total or free T3 level is normal in subclinical hypothyroidism.
 Sarcoidosis results in hypercalcemia (increased 1,25-dihydroxyvitamin D level from a1-hydroxylae activity).
 Increased active vitamin D (1, 25 dihydroxyvitamin D) results in increased gut absorption of calcium, increased bone
resorption of calcium, and decreased calcium/phosphate excretion by kidneys.
 Elevated calcium and low PTH indicates non-PTH mediated hypercalcemia (normally PTH causes increase Ca).
Differential includes: cancer related hypercalcemia (2/2 osteolytic lesions of bone), humorally mediated secretion of
PTHrP from tumor cells, or granulomatous diseases.
 CASR (G-coupled protein calcium-sensing receptor) gene mutation will result in hypercalcemia. These genes code for
receptors in parathyroid gland and kidneys. The sensor mutation shifts the receptor in upper limit of calcium.
 HCTZ: thiazide diuretic: reduces blood volume by acting on kidneys to reduce sodium reabsorption in DCT. Increases
reabsorption of calcium in DCT via action on NaCl co-transporter.
 Presentation of Cushing syndrome: weight gain, facial hair, alopecia, amenorrhea, fatigue, central adiposity, buffalo
hump, hypertension, round facies, acne, purple striae, supraclavicular fat deposition.
 Differential of Cushing Syndrome: Cushing disease (pituitary adenoma), adrenal tumor cortisol production, ectopic
ACTH production, excess glucocorticoids (iatrogenic Cushing syndrome.
 Hypokalemia in DKA must be corrected prior to starting insulin infusion to prevent arrhythmia (if K <3.3).
 Test for secondary causes of bone loss prior to starting bisphosphonate therapy (CBC for malignancy, TSH, P7 for
CKD and calcium levels, urine calcium for hypercalciuria, PTH level, and 25-hydroxy vitamin D).
 Manage heavy menstrual bleeding and hirsutism in PCOS with combination OCP. OCP prevents unopposed estrogen
induced proliferation of endometrium, and suppresses excess androgen production.
 PCOS: patients remain in stagnant follicular stage, resulting in unopposed estradiol secretion from small ovarian
follicles, which cause proliferation of endometrium in the absence of progesterone secretion from the corpus luteum.
Ultimately causes endometrial hyperplasia and heavy menstrual bleeding from anovulatory bleeding.
 Intraovarian androgen production is increased in PCOS: results in hyperandrogenism and hirsutism.
 Progestin therapy alone will not suppress androgen production and treat hirsutism. It will only treat the bleeding.
 Metformin in PCOS: increases insulin sensitivity, and reduces serum free testosterone, but not as effective as OCP.
 Adrenocortical carcinoma (ACC): surgical excision. Causes Cushing syndrome: round red face, buffalo hump,
violaceous striae, acanthosis nigricans, ecchymosis. Elevated 24 hour urine cortisol on repeated measurements. Low
ACTH (consistent with ACTH independent cause). Radiographic characteristics of ACC: large mass, irregular
borders, calcification, high attenuation (high Hounsfield units) on CT, delay in contrast medium washout (<50% at 10
minutes). Even after resection, adjuvant mitotane (adrenal cytotoxic med) may be needed. FNA cannot distinguish
benign adenoma from carcinoma. Not used for evaluation of ACC. Radiation therapy may be used if metastatic.
 Hypocalcemia: long QT, tremulousness, shakiness, muscle twitching,
 Hypomagnesemia impairs parathyroid hormone secretion, which leads to hypocalcemia. Low magnesium is also
associated with resistance to PTH activity at the bone. Magnesium must be replete prior to serum calcium correction.
 Normally, hypocalcemia stimulates PTH production; however, hypomagnesemia impairs PTH release and function.
 1,25-dihydroxyvitamin D has a short half-life. Its measurement only reflects active levels of vitamin D.
 25-hydroxy vitamin D has a longer half-life and more accurately reflective of total body stores of vitamin D.
 Diabetes goals: A1C <7%, preprandial sugars 70-130, and postprandial sugars <180 (1-2 hours after meals).
 Give lifestyle 4-6 weeks to improve glycemic control. Otherwise, change meds q3 months until at goal.
 Dapagliflozin: sodium glucose transporter 2 inhibitor (SGLT-2): increases excretion of glucose via kidneys.
 Glipizide: sulfonylurea: stimulates insulin secretion from pancreatic beta cells. Induces weight gain.
 Sitagliptin: dipeptidyl peptidase 4 inhibitor (DPP-4): slows gastric emptying and suppresses glucagon secretion.
 Rule out hemachromatosis as a cause for hypogonadism. Workup with serum transferrin saturation and ferritin.
Patients with arthralgia, hepatomegaly, bronze diabetes.
 Hypogonadism: low testosterones, low LH, low FSH, diminished libido, erectile dysfunction.
 Causes of hypogonadotropic hypogonadism: infiltrative (hemochromatosis, sarcoidosis), metastatic cancer to pituitary
gland, lymphoma.
 Klinefelter syndrome: 47XXY. Hypergonadotropic hypogonadism. Elevated LH and elevated LSH.
 40% of calcium in serum is bound to albumin. Pseudohypercalcemia/Factitious hypercalcemia: increased protein
binding of calcium in hyperalbuminemia (2/2 severe dehydration), and paraproteinemia (MM). check ionized Ca.
 Measure 1,25-dihydroxy vitamin D level in assessing PTH vs non-PTH mediated hypercalcemia (excess Vit D).
 MM: tumor induced osteoclast mediated bone resorption 2/2 cytokines released by myeloma cells: hypercalcemia that
is non-PTH mediated, and not due to excess Vitamin D levels.
 Prior to starting androgen/testosterone therapy for hypogonadism, patient’s desire for fertility should be explored.
Testosterone replacement may cause decrease spermatogenesis and infertility. Exogenous testosterone suppresses
both GnRH and FSH/LH production, which depletes intratesticular testosterone. Symptomatic men with reduced
libido, erectile dysfunction, mood changes, irritability, fatigue, and memory loss.
 Thyroid storm may be precipitated by high iodine load from dye contrast in setting of undertreated Graves. Other
triggers: infection, surgery, MI, trauma, parturition. S/S: fever, tachycardia, CHF, GI dysfunction, AMS. Treatment is
supportive care, reduction of thyroid hormone production, decreasing peripheral conversion of T4 to T3, addressing
adrenergic and thermoregulatory symptoms, and treating precipitating factors. Tx with thionamines and BB.
 Euthyroid sick syndrome: low TSH, and low thyroid hormone levels.
 Postsurgical hypoparathyroidism s/p parathyroidectomy. In patients without parathyroid function, stimulation of renal
conversion of 25-hydroxyvitamin D (from liver) to active form of 1,25-dihydroxy vitamin D (calcitriol) is lost, as is
the signal to increase calcium resorption in DCT and LOH. As a result, both calcitriol and calcium supplementation
must be provided. However, without resorption of calcium by kidneys, the repletion of calcium will result in
hypercalciuria. Recommended 24 hour urine calcium goal should be <300 mg/24 hours with serum calcium in low
normal range of 8.0-8.5. if urine calcium levels are >300, then decrease oral calcium intake. If the urine calcium is
>300 and serum calcium also >8.5, then decrease both calcium and calcitriol supplementation.
 Usually 25-hydroxyvitamin D is best indicator of total body vitamin D stores; but in abnormal parathyroid function
that requires treatment with activated vitamin D (1,25 dihydroxy), you need to measure active 1,25 vitamin D levels.
 For exercise induced hypoglycemia, continue basal dose, but decrease prandial insulin amounts. Exercise is known to
increase glucose utilization by muscles, which induces hypoglycemia in setting of exogenous insulin administration.
Don’t stop the basal insulin, because it may lead to DKA (hyperglycemia and insulin deficiency in DM1 will increase
release of counterregulatory hormones and cause deceased lipolysis and oxidation of FFA).
 When hypothyroidism is suspected despite normal TSH, order T4 levels, especially if patient has had loss of
hypothalamic pituitary function from radiation therapy. Normal TSH in context of clinical hypothyroidism suggests
central hypothyroidism. TSH is inaccurate.
 S/S hypothyroidism: fatigue, constipation, periorbital edema, brittle hair, lateral eyebrow loss, cold intolerance, dry
skin, delayed DTR uptake, anemia, hyponatremia.
 Distinguish primary hyperparathyroidism and familial hypocalciuric hypercalcemia (FFF) with 24-hour urine calcium
and creatinine, which establishes amount of renal Ca wasting and evaluates the urine calcium-creatinine ratio. If total
urine calcium <200mg/24h and calcium-creatinine ratio <0.01, likely FHH diagnosis, which is from mutation in
specific calcium sensing receptor in parathyroids and kidneys, resulting in hypercalcemia. FHH rarely causes severe
hypercalcemia warranting therapy to lower serum levels. Screen family members.
 Fasting glucose target for DM: 70-130
 Preprandial glucose target for DM: 80-130.
 Postprandial 2 hour glucose target for DM: <180.
 Prandial insulin should be given immediately prior to meals.
 ACTH independent Cushing syndrome 2/2 benign adenoma: Diagnose via CT scan with R adrenal mass c/w benign
adenoma features: low attenuation on unenhanced CT scan w/ density <10 Hounsfield units, and rapid washout of IV
iodine contrast material (>50% at 10 minutes). Manage postop adrenalectomy patients with steroids; these patients
may develop acute adrenal failure (hypocortisolism) from hypothalamic-pituitary-adrenal (HPA) axis suppression and
contralateral adrenal atrophy. Treat them with stress dose steroids with long and slow taper until HPA axis recovery is
confirmed. On average, most patients have adrenal insufficiency up to 12 months.
 Clinical features of ACTH independent Cushing syndrome 2/2 adrenal adenoma: elevated urinary free cortisol, lack of
suppression of serum cortisol following low-dose overnight dexamethasone suppression test. HTN, weight gain,
weakness, easy bruising.
 Adrenal cancer features: large lesions (>4-6 cm) with irregular margins and areas of necrosis and calcification.
Unenhanced CT demonstrates high attenuation (density >10 Hounsfield units), and washout of IV iodine contrast
media <50% at 10 minutes. Treat postoperatively with mitotane, an adrenolytic drug for adjuvant therapy in patients
with locally persistent or metastatic adrenocortical carcinoma (ACC). Mitotane achieves objective remissions in 25%
of patients.
 Preoperative phenoxybenzamine: management of pheochromocytoma. Alpha blockade decreases risk of CV
complications related to excessive catecholamine release during intraoperative manipulation of the tumor.
 Paget disease of bone: focal areas of accelerated bone remodeling that ultimately causes bony overgrowth, which
impairs the integrity of the affected bone. Skull, spine, pelvis, lower extremity long bones. Radiographic findings of
bone turnover: concurrent osteolytic and osteoblastic changes: thickened cortical bone and coarsening of the
trabecular bone of the femur. Treat with antiresorptive therapy if hypercalcemic or symptomatic with pain (usually
2/2 increased bone metabolic activity). Antiresorptive agents: nitrogen-containing bisphosphonates (alendronate,
pamidronate, risedronate, zoledronic acid). Bisphosphonates stabilize bone turnover via suppressing bone resorption
and new bone formation; reduces serum alkaline phos levels.
 Most patients with Paget disease of bone presents asymptomatically with elevated serum alkaline phosphatase.
 Osteoblastic disease process: metastatic disease to bone
 Osteolytic disease process: MM. UPEP/SPEP.
 For pregnant patients with hypothyroidism, the levothyroxine dose should be increased, to lower the serum TSH level
since maternal thyroid hormone production increases by 30-50% during pregnancy. Important to increase
levothyroxine to provide adequate thyroxine (T4) for the neurologic development of the fetus. Pregnancy causes
physiologic increase in serum total T4 level and decrease in serum TSH level (from rise in serum HCG level); both
HCG and TSH share sequence homology in their alpha subunit. As HCG levels rise in pregnancy, the hormone can
bind to TSH receptors, causing a reduction in serum TSH. Consequently, TSH range shifts to lower values in
pregnancy from 0.5-5 to 0.03-2.5 (during first trimester). Increase of serum total T4 1.5x normal nonpregnant
reference range is 2/2 high levels of estrogen which causes increased serum total protein levels (including thyroid
hormone binding globulin (THBG). In pregnant patient in first trimester, TSH>2.5 requires increase in levothyroxine.
Recheck TSH in 4 weeks, and after each subsequent trimester.
 Acute adrenal failure presents with symptomatology from acute cortisol and aldosterone deficiency including GI
disturbances (nausea/vomiting, abdominal pain), lethargy, lightheadedness, weakness, hypotension, shock,
hypoglycemia, and lyte disturbances (hyponatremia and hyperkalemia). Think about adrenal hemorrhage in setting of
sudden H/H drop if on anticoagulation therapy, postoperatively, or with abnormal hemostasis (HIT, APLAS), and
sepsis. Treat with stress dose steroids (Hydrocortisone 50-100 IV q6-8h) and supportive care (IVF, pressors, etc).
 Chronic long-acting opioid dependence: common cause of hypogonadotropic hypogonadism and potential etiology of
secondary adrenal insufficiency. Doesn’t cause hyperkalemia, because mineralocorticoid secretion is preserved.
 Pituitary apoplexy: acute hemorrhage into pituitary. May cause secondary adrenal insufficiency. Also p/w headache,
vision changes. Does not result in hyperkalemia (secondary adrenal insufficiency, not primary).
 Autoimmune adrenalitis: may cause primary adrenal failure. Associated with other autoimmune diseases. Onset of
symptoms usually more gradual, with skin hyperpigmentation.
 Pituitary microadenomahigh ACTHhigh cortisol level. Cushing syndrome: weight gain, muscle weakness,
metabolic syndrome, amenorrhea, loss of scalp hair, acne, cushingoid haibtus, violaceous striae on abdomen and the
back of the arms, hirsutism.
 Evaluate a new hypercortisolism patients for osteoporosis. With DEXA scan. Cushing patients have decreased
intestinal calcium absorption, decreased bone formation, increased bone resorption, and decreased renal calcium
reabsorption.
 8mg dexamethasone suppression test helps localize the source of ACTH (normal 10-50) in patients who present with
Cushing syndrome. Pituitary Cushing disease responds to 8mg dex with a suppressed AM cortisol level (normal 5-
25). Ectopic ACTH production does not respond to the 8mg of dex, and AM cortisol levels remain elevated. This test
may have false positive results. Intrapetrosal sinus sampling has higher Sp/Sn when completed by skilled
interventional radiologist.
 Cushing disease is not malignant; usually caused by pituitary adenoma. Cushing syndrome 2/2 ectopic ACTH is
usually caused by a malignancy/cancer. PET may localize the source of ectopic ACTH.
 How to do the dexamethasone suppression test: measure cortisol at 8pm, give 8mg dexamethasone after, then recheck
AM cortisol level the next morning.
 24-hour urine free catecholamine and metanephrine levels: screening for pheochromocytoma. Screen for pheo prior to
surgery for patients with adrenal lesions.
 Primary adrenal failure 2/2 autoimmune adrenalitis management: prednisone 5mg daily, fludrocortisone 0.05mg daily.
Failure in production of all hormones within the adrenal cortex (requires mineralocorticoid and glucocorticoid
replacements). Okay to give physiologic replacement in setting of clinical stability and in absence of acute illness.
 Hydrocortisone: has both glucocorticoid and mineralocorticoid properties; primarily glucocorticoid activity at
physiologic replacement doses of 12.5mg-25mg bid/tid. If >50mg total daily dose, Hydrocort has adequate
mineralocorticoid activity to allow its use as monotherapy for physiologic supplementation doses, but will have supra-
physiologic amounts of glucocorticoid activity (may lead to iatrogenic Cushing syndrome if long term use).
 Stress dose steroid doses. For minor stress (URI, fever, minor sx): 2-3x basal dose of hydrocortisone. For moderate
stress (moderate surgery with GA): 45-75mg/day. Major stress (ICU, trauma, major surgery, L&D): 150-200mg/day
with gradual taper following resolution of stress.
 Primary thyroid lymphoma, occurs in elderly women with longstanding hx of Hashimoto thyroiditis. Clinically
presents as rapid onset (weeks) enlarging goiter with weight loss and night sweats. CT shows a diffusely enlarged
thyroid gland (“doughnut sign”-enlarged thyroid extends behind and completely encircles the trachea). Dx via biopsy
of thyroid with flow cytometry. Treat with chemo and radiation therapy.
 Hashimoto Thyroiditis: Chronic lymphocytic thyroiditis: autoimmune condition where thyroid gland is destroyed via
antithyroid antibodies. Autoantibodies against thyroid peroxidase, thyroglobulin, and TSH receptors. Occurs via CD8
T cell activation in response to a cell mediated immune response which destroys thyrocytes. Treat with thyroid
replacement (levothyroxine).
 Graves disease: toxic diffuse goiter: autoimmune disease of hyperfunctioning and enlarged thyroid gland. Mechanism
occurs via thyroid stimulating immunoglobulin (TSI) antibody which causes thyroid gland to produce excess thyroid
hormone, causing thyrotoxicosis.
 S/S of hyperthyroidism: irritability, muscle weakness, sleeping problems, tachycardia, poor tolerance of heat,
diarrhea, weight loss, pretibial myxedema, proptosis.
 S/S of hypothyroidism: apathy, fatigue, weight gain, pale/puffy face, feeling cold, joint/muscle pain, constipation, dry
and thinning hair, heavy menstrual flow/irregular periods, depression, panic disorder, bradycardia, difficulty getting
pregnant and maintaining pregnancy.
 Papillary thyroid cancer: very slow growth, thyroid not typically diffusely enlarged; usually with a distinct nodule and
potentially concomitant cervical lymphadenopathy.

Subacute (de Quervain) thyroiditis is associated with acute onset of anterior neck pain. It is typically
seen following a viral illness in the preceding months. The changes on CT are typically a patchy
infiltrate with minimal lymphadenopathy. This patient's image reveals marked diffuse enlargement of
the thyroid, and she did not have a history of prior illness.

MKSAP: HEMATOLOGY & ONCOLOGY


 TTP: thrombotic thrombocytopenic purpura: MAHA (schistocytes, LDH, thrombocytopenia, indirect
hyperbilirubinemia), fever, renal dysfunction (hematuria, proteinuria), AMS. Treat with plasmapheresis. Send off
ADAMS-TS13 assay for inhibitor and titer levels (use for prognosis, not diagnosis). A low activity and high inhibitor
titer signifies poor prognosis and higher risk of relapse. TTP may overlap with HUS (hemolytic uremic syndrome),
seen in kids, associated with E. coli O157:H7 or Shigella (diarrheal illness). Same tx for both. HUS causes renal
failure via bacterial toxin that imitates antigen on renal endothelial cells, which causes renal cell death.
 Indirect bilirubinemia seen in hemolysis.
 Hereditary spherocytosis: osmotic fragility test.
 4-factor prothrombin complex concentrate (4F-PCC): inactivated form of factors 2, 7, 9, 10. Use for warfarin reversal
in acute bleed. Less risk of volume overload, and acts quicker than FFP, and doesn’t need to be thawed.
 Chronic thromboembolic pulmonary hypertension: patients with hx of PE. Evaluate initially with VQ scan to rule out.
However, if +CTEPH, then additional workup: RHC with PA pressure and pulmonary arteriography may need to be
done. Mean PA pressure >25, normal wedge pressure. Usually appears within 2 years after PE. CTA is not sensitive
enough for chronic PE, because CTEPH involves chronic changes in the pulmonary vasculature 2/2 organization of
thrombus and recanalization and is not associated with distinct intraluminal filling defects. Similarly, D dimer also not
sensitive enough.
 PNH: paroxysmal nocturnal hemoglobinuria. Acquired clonal stem cell disorder. Dark urine, hypocellular marrow,
unprovoked atypical thrombosis, pancytopenia/hemolytic anemia, flow cytometry shows absent CD55 (DAF: decay
accelerating factor) and CD 59 (MIRL: membrane inhibitor of reactive lysis) on peripheral RBC/WBC (which are
complement regulatory proteins). Caused by mutation in PIG-A gene, leading to reduction/absent
glycosylphosphatidylinositols (RBC anchoring protein).
 The myelodysplastic syndromes (MDS) are clonal hematopoietic stem cell disorders characterized by ineffective
hematopoiesis. MDS usually presents with anemia or pancytopenia and a hypercellular marrow with dysplastic
changes in cell precursors.
 Myeloproliferative neoplasms (MPNs) can present with splanchnic thrombosis and should be considered in the Ddx
of unusual thrombosis. Peripheral blood counts have elevated cell lines, and the marrow in MPNs is typically
hypercellular or fibrotic.
 Budd-Chiari syndrome (BCS): acute portal HTN caused by thrombosis of the hepatic veins. May clinically present
with fulminant hepatic failure, or subacute decompensation with ascites and tender hepatomegaly. Up to 50% of
patients with idiopathic BCS have an acquired mutation in JAK2, without overt suggestion of a MPN. JAK2 testing is
part of routine diagnostic protocol that includes consideration of PNH in the differential diagnosis of splanchnic vein
thrombosis.
 Portal venous system directs blood from mesentery to the liver. Blood from liver drains into hepatic vein, which then
flows to the IVC.
 Sublingual and suppository bypass the portal venous system (first pass effect), because the 1/3 rectum is drained into
portal system, while 2/3 drains into internal iliac into IVC.
 Hemophilia A: factor 8 deficiency. X linked recessive. Prolonged PTT that fully correct in a mixing study (ddx: factor
8, 9 (autosomal recessive), 11 deficiency).
 The bleeding time tests primary hemostasis rather than fibrin formation. It is prolonged in patients with platelet
dysfunction, vWD, thrombocytopenia, and anemia.
 Frequent hypoglycemic events and hypoglycemic unawareness increases risk of morbidity/mortality from recurrent
hypoglycemia; glycemic goals should be individualized to account for patient specific factors, including age and
comorbidities. ADA suggests A1C <8% if patient has decreased life expectancy, hx of severe hypoglycemia, multiple
comorbidities or advanced microvascular or macrovascular disease. Implement less stringent glycemic goals.

MKSAP: RHEUMATOLOGY
 Treat hyperuricemia with febuxostat in a patient with hypersensitivity reaction to allopurinol. Repeated gout attacks or
flares is defined as: >2 attacks per year, >1 attack in setting of CKD, tophaceous deposition on exam/imaging, or hx
of urolithiasis. These patients require urate lowering therapy (1st line is allopurinol). Febuxostat: new nonpurine
noncompetitive xanthine oxidase inhibitor. Low dose ASA may increase serum urate 2/2 effects on renal uric acid
transport. ACR does not recommend d/c ASA in patients with indications (CAD).
 In absence of active disease, ppx with colchicine should be continued for the greater of the following: 6 months, 3
months after achieving target serum urate level if without tophi, 6 months after achieving serum urate level with
resolution of tophi. Probenecid: uricosuric agent (promotes renal excretion of uric acid): viable alternative first line
urate lowering agent if contraindicated to xanthine oxidase inhibitor therapy. Probenecid increases risk of kidney
stones, and contraindicated if hx of stones.
 Tophi: reservoirs of monosodium urate deposition.
 Anti-inflammatory prophylaxis to prevent gout attacks is recommended when urate lowering therapy is initiated,
because of the paradoxical increased risk of acute gout attack when serum urate levels are rapidly decreased by the
medications. Ppx should be continued in presence of any active disease (tophi or flares). Colchicine is first line.
 Pegloticase: IV synthetic uricase replacement for treatment-failure gout. It is immunogenic, and patients usually
develop antibodies to the medication, which leads to reduced effectiveness and increased risk of hypersensitivity rxn.
 Uricosuric drugs: Probenecid, and sulfinpyrazone: promotes kidney clearance of uric acid via inhibition of urate-anion
exchangers in the PCT responsible for urate reabsorption. Contraindicated for CKD or with risk of kidney stones.
 Jaccoud Arthropathy: SLE arthritis, which is nonerosive, but persistent periarticular inflammation that affects the
structural integrity of the joint capsule/supporting ligaments. Characterized by reducible subluxation of the digits,
swan neck deformities, and ulnar deviation 2/2 attenuation of joint supportive structures. Seen in 5% of SLE,
confused with RA. Also seen in scleroderma, MCTD, Sjogren. Radiographs negative for erosive disease.
 Joint hypermobility: ability to painlessly move a joint beyond normal range of motion.
 Hypermobility syndrome: MSK pain and generalized joint hypermobility in otherwise healthy individuals.
 MCTD: mixed connective tissue disease: systemic sclerosis, polymyositis, SLE, Raynaud phenomenon, hand edema,
puffy fingers, synovitis. High titers of anti-U1-RNP Ab (ribonucleoprotein).
 RA: nonreducible hand deformities: severe erosive changes on radiographs and periarticular osteopenia.
 Diagnosis of RA: anti-CCP Ab (cyclic citrullinated peptide) and RF serum tests. Affecting proximal IP and MCP
joints of fingers, wrists, and feet joints. Prolonged morning stiffness.
 RF positivity is also with RA, infection, Hep C, and other autoimmune disorder.
 SLE: +ANA, alopecia, aphthous ulcers, malar rash, pericarditis, pleuritis, serositis, cytopenias.
 Amyopathic dermatomyositis: heliotrope eruption in form of violaceous periorbital rash, Gottron papules over
extensor surfaces of small joints of the hands, photodistributed violaceous poikiloderma (V sign, shawl sign).
Amyopathic because of the negative clinical/serum/EMG findings of muscle involvement/myositis. May be triggered
by sunlight exposure; associated with underlying malignancy. Tx: steroids, immunosuppression.
 Amyopathic dermatomyositis is 20-25% of patients with dermatomyositis. Check muscle enzymes (CK, aldolase,
AST, ALT, LDH), EMG, muscle biopsy, even in absence of muscle weakness.
 Polymorphous light eruption (PMLE): dermatologic condition, where patients develop skin lesions after exposure to
sunlight; lesions last several days, resolve spontaneously in absence of re-exposure. May see urticarial wheals,
papules, plaques, vesicles. Usually in spring, and triggered by intense sun exposure.
 Rosacea: chronic inflammatory condition, causes acneiform eruption and flushing on face. Two types: vascular and
papular pustular rosacea. Vascular presents as persistent flushing on central face with prominent telangiectasias.
Pustule and papules are seen in the inflammatory variant, but are not follicular based (in contrast to acne).
 For OA: studies have shown that effusion in joint, withdrawal of fluid from knee, severity of disease, absence of
synovitis, injection under US guidance and greater symptoms at baseline will all improve likelihood of response to
intra-articular steroid injection.
 Intra-articular steroids are good for <2 joints in acute gout flare if PO NSAIDs are contraindicated. Gout flare may be
triggered in setting of sepsis with fever and dehydration. Colchicine is metabolized by hepatic CYP3A4 enzyme,
which is inhibited by clarithromycin (if patient is on triple/quadruple therapy). Co-administration of colchicine and
clarithromycin risks colchicine toxicity and death. CKD makes NSAIDS undesirable because of COX inhibition.
 Severe adult onset HSP (Henoch-Schonlein Purpura): small vessel vasculitis affecting skin, joints, kidneys, and GI
tract. Leukocytoclastic vasculitis and IgA deposition on skin biopsy (palpable purpuric lesions on lower extremities,
including the soles of the feet); treat with steroids. Abdominal US may reveal thickening and edema of ileum. Usually
onset is >10 days after URI symptoms, with arthritis. UA shows inflammatory nephritis with mixed casts. +FOBT.
 Cyclophosphamide: alkylating agent, potent immunosuppressant. Tx severe SLE and ANCA associated vasculitis.
 Dapsone: Abx that has anti-leukocyte activity. Occasionally treats leukocytoclastic vasculitides, including HSP.
 Don’t mix two biologic DMARD (disease modifying antirheumatic drugs) together; higher risk of infection. Try
nonbiologic leflunomide (Arava) for chronic moderate-severe RA with >1 poor prognostic markers (young age, >3
joints, seropositivity, elevated inflammatory markers, +radiographs). If active disease despite sulfasalazine and
etanercept (Enbrel), with joint inflammation and synovitis, then initiate “treat to target” approach, to add biologic
agent (preferably a TNF inhibitor, abatacept, anakinra, rituximab) w/ nonbiologic DMARD (MTX, leflunomide, etc).
 Fibromyalgia: chronic widespread pain, tenderness of skin/muscles to pressure (allodynia), fatigue, sleep disturbance,
exercise intolerance, lack of response to multiple meds. Associated with IBS, irritable bladder, pelvic pain,
vulvodynia, headache, TMJ, chronic pain syndrome. >3 months, with cognitive difficulties as well.
 For SLE who is positive for anti-Ro/SSA or anti-La/SSB antibodies, counseling should be done regarding congenital
heart block in their children (2% risk). Some infants require pacing from birth if there is complete heart block at
delivery. She has higher risks of miscarriages, stillbirth, preeclampsia, and premature delivery. Experts recommended
conception when SLE has been quiescent for >6 months. Hydroxychloroquine is safe in pregnancy and has been
shown to reduce risk of congenital heart block in newborns whose mothers are positive for anti-Ro/SSA, anti-La/SSB.
 Anti-Ro/SSA Ab increases risk of developing subacute cutaneous lupus.
 Hemochromatosis causes secondary osteoarthritis, involving 2nd and 3rd MCP joints with hook-shaped osteophytes on
radiographs of hands. AR disorder, increased iron absorption from gut. 40-60% develops arthropathy. Elevated
transferrin and ferritin levels on serum. Liver biopsy and genetic testing to confirm: homozygous C282Y mutation of
the HFE gene. Transferrin >55% is diagnostic.
 Serum AFP (alpha-fetoprotein): elevated in liver disease (acute/chronic viral hepatitis, HCC), other cancers.
 Diagnose spondyloarthritis with MRI of SI joints. Chronic inflammatory back pain in patients <45 yo. Important to
diagnose early on, because of monitoring for development of cardiovascular and other major organ damage. HLA-
B27 is supportive of the diagnosis, but negative result does not rule it out. XR may show sacroiliitis (erosive changes,
and sclerosis).
 IV technetium 99m binds to hydroxyapatite crystals; increased uptake reflects increased bone turnover 2/2 infection,
cancer, trauma, and arthritis. Nonspecific for bone/joint/periarticular disease.
 Limited cutaneous systemic sclerosis (LcSSc): isolated distal skin thickening (face, neck, hands distal to wrists), not
accompanied by internal organ fibrosis, associated with PAH (dx with TTERHC); rule out if symptomatic.
 In patients with polymyositis with positive anti-Jo1 antibody and features of antisynthetase syndrome, get a diagnostic
CXR to rule out ILD. Other pulmonary findings associated with polymyositis/dermatomyositis: ILD, hypoventilation
2/2 weakness of respiratory muscle, aspiration PNA, and PAH. ILD is strongly associated with +autoantibodies to
transfer RNA synthetases including anti-Jo 1 antibodies. Various patterns of ILD occur: nonspecific interstitial
pneumonitis-NSIP (most common), usual interstitial pneumonia-UIP, bronchiolitis obliterans organizing pneumonia-
BOOP. Ultimately, the pattern of involvement determines steroid responsiveness and prognosis.
 Myocarditis presentation: fatigue, chest pain, CHF, cardiogenic shock, arrhythmia, and sudden death.
 Monitor lipid profile in RA patients on tofacitinib (biologic DMARD). May cause drastic increase in all lipid lines. It
may also lower leukocyte counts, causing lymphopenia, neutropenia, and anemia with long term use.
 Serum urate target goal is <6 for gout. Gradually titrate allopurinol dose until urate is at goal (minimizes risk of
hypersensitivity reaction). Start at 100 mg/d (or 50 mg/d in CKD), and titrate upwards q2-5 weeks). Don’t discontinue
losartan if they are on it for CKD/HTN, as it has some uricosuric effects and may be helping lower serum urate.
 PMR is in 50% of patients with TA/GCA. Tx TA/GCA with high dose steroids 60mg/day or 1mg/kg/day. Low dose
steroids 10-20 mg/d is only for PMR alone.
 Screen for LTBI with IGRA/TST/PPD prior to starting biologic therapy with TNF-a inhibitor (DMARD). IGRA is
more costly but may be more sensitive in patients already on immunosuppressive therapy.
 Nonbiologic DMARD: MTX (Rheumatrex), sulfasalazine (Azulfidine), hydroxychloroquine (Plaquenil), leflunomide
(Arava), gold salts
 Biologic DMARD: adalimumab (humira), abatacept, (Orencia), anakinra (Kineret), etanercept (Enbrel), golimumab
(Simponi), infliximab (Remicade), rituximab (Rituxan), tocilizumab (Atlizumab)
 TNF-a inhibitors inhibit formation of granuloma, high risk of reactivation of TB.
 CVID: common variable immunodeficiency: very low IgG, and IgA/IgM are relatively low. Adults/children develop
chronic lung disease, malabsorption, recurrent infections, lymphoma, and autoimmune d/o such as RA. They also
have recurrent sinopulmonary infections, ear infections, conjunctivitis.
 Distinguish inflammatory vs noninflammatory joint pain: former is relieved with NSAID, latter (DJD/DDD) isn’t.
 Psoriatic arthritis: enthesitis, dactylitis, tenosynovitis, arthritis of DIP joints, asymmetric oligoarthritis, spondylitis.
HLA-B27 antigen may be positive if +axial involvement.
 Reactive arthritis: Reiter’s syndrome. Postinfectious arthritis 2/2 GU/GI (urethritis, diarrhea). Usually asymmetric
monoarthritis/oligoarticular arthritis in lower extremities, several weeks after infection. Not uncommon for stool/urine
to be sterile at time of diagnosis. HLA-B27 may be positive. Enthesopathy (inflammation at site where ligaments,
tendons, joint capsule or fascia attaches to bone), dactylitis, sacroiliitis may occur. Erosive disease is uncommon. 30%
will have uveitis or conjunctivitis.
 Sjogren syndrome manifests as inflammation of exocrine glands (salivary glands, lacrimal glands, pancreatic exocrine
glands). Higher risk of lymphoma (diffuse B cell lymphoma, and MALT lymphoma), up to 44x general population.
Chronic B cell activation leads to development of a clone of malignant B cells. Hypocomplementemia, splenomegaly,
lymphadenopathy, gammopathy, skin vasculitis, and cryoglobulinemia predict the development of lymphoma. Biopsy
shows focal centrilobular collections of lymphocytes.
 Lymphoma is associated with leukocytoclastic vasculitis: lower extremity scattered palpable purpura.
 Seronegative peripheral/axial spondyloarthritis: HLA-B27 testing. Uveitis, enthesitis of achilles tendon, psoriasis,
IBD, infection, sacroiliitis. Beware: HLA-B27 is positive in 5% of population; may lead to false positive.
 ANCA associated vasculitides: Wegener granulomatosis (granulomatosis with polyangiitis), microscopic polyangiitis,
churg-strauss syndrome (eosinophilic granulomatosis with polyangiitis).
 Wegener’s: systemic necrotizing vasculitis that affects upper/lower respiratory tract and kidneys. 70% of patients
have upper airway manifestations (sinusitis, nasal/inner ear/laryngotracheal inflammation). Scleritis (inflammation of
fibrous layer of the eye underlying the conjunctiva and episclera). Nephritic syndrome w/ HTN, GN, casts.
 Scleritis may be 2/2 autoimmune disease such as RA, relapsing polychondritis, IBD, or vasculitis.
 Behcet syndrome: recurrent oral/genital ulcers, eye/skin involvement, and pathergy. Uveitis.
 Sarcoidosis: multisystem disease. granulomas in lung tissues. Uveitis.
 Microscopic polyangiitis: necrotizing vasculitis predominantly affecting lungs and kidneys.
 Familial Mediterranean fever (FMF): AR disorder associated with mutation of MEFV1 gene. Clinical features: fever,
polyserositis (relapse/remitting abdominal serositis), inflammatory arthritis, erysipeloid rash around ankles (mimics
cellulitis), elevated acute phase reactants. Attacks lasts 1-3 days, usually self limited, but may be dramatic; sometimes
the inflammatory episodes occur independent of the febrile episodes. AA amyloidosis is potential long-term
consequence of FMF 2/2 production and accumulation of serum amyloid A. colchicine: affects function of
inflammatory cells that play a role in the cytokine overproduction in FMF. Tx with colchicine. FMF genetic mutations
affect function of pyrin, which results in over activation of the inflammasome (important constituent of the innate
immune system responsible for IL1 production. Dysregulation of the inflammasome results in overproduction of IL1.
Anakinra is IL-1 inhibitor; used for patients unresponsive to colchicine.
 OA refractory to intra-articular steroid injections, and cannot take NSAIDS: SNRI (duloxetine, venlafaxine).
 Treat NSIP-ILD associated diffuse cutaneous systemic sclerosis (DcSSc) with cyclophosphamide, to help calm the
inflammation. High dose steroids are frequently used in these patients but have unclear benefit and may precipitate
scleroderma renal crisis. Azathioprine may have a role as maintenance therapy.
 RF gout: old, man, HTN, DM, obesity. Podagra=gout of big toe. Dx with joint aspiration, identifying monosodium
needle-shaped urate crystals within leukocytes via synovial fluid analysis.
 Always send joint aspirates/fluid studies for gram stain/culture, leukocyte count, and crystal analysis.
 Seronegative spondyloarthropathies: negative RF: ankylosing spondylitis, reactive arthritis, psoriatic arthritis, etc.
 Psoriatic arthritis: pitting nails, onycholysis, psoriatic nail dystrophy, enthesitis, dactylitis, tenosynovitis, arthritis of
DIP, asymmetric oligoarthritis, spondylitis, psoriasiform skin lesions (umbilicus/gluteal cleft/extensor surfaces/
posterior auricular region/scalp)
 Eosinophilic granulomatosis with polyangiitis: EGPA: Churg-Strauss Syndrome: eosinophilia, migratory pulmonary
infiltrates (capillaritis w/ hemoptysis), palpable purpuric skin rash, mononeuritis multiplex in the setting of antecedent
atopy. 40% of patients with EGPA are negative for ANCA, 60% has +p-ANCA (antimyeloperoxidase). Hx of asthma,
high IgE levels. Sural nerve tissue biopsy shows necrotizing vasculitis with eosinophilic granulomas.
 Polymyositis: elevated muscle enzymes (CK, aldolase, LDH, AST/ALT), symmetric progressive proximal muscle
weakness, EMG abnormalities (increased insertional activity, spontaneous fibrillations, polyphasic motor unit
potentials in proximal muscles), MRI (inflammatory changes in muscle), absent skin findings, muscle biopsy is
diagnostic. Tx with prednisone (1mg/kg/d), for severe disease, may give IV solumedrol, or MTX, azathioprine for
steroid sparing protocol. Treat refractory/recurrent disease with mycophenolate mofetil, IVIG, rituximab,
cyclophosphamide, or TNFa-inhibitors.
 Cyclosporin: immunosuppressant, preferentially targets T cells. Treats RA, SLE, inflammatory myositis, psoriasis,
pyoderma gangrenosum, and IBD. Toxicity is common: hypertension, nephrotoxicity, tremor, hirsutism.
 Leflunomide: as effective as MTX for RA. Toxicity includes liver and hematopoietic abnormalities, infection, ILD.
 DISH: diffuse idiopathic skeletal hyperostosis: flowing osteophytes involving anterolateral T spine, >4 contiguous
vertebrae with preservation of intervertebral disk space (no loss of height) and absence of apophyseal joint or SI
inflammatory erosive changes. Flowing candlestick wax dripping appearance on T spine lateral XR. May occur with
or without OA or inflammatory arthritis (bony hypertrophy of interphalangeal joints in hands); separate finding of
calcification and ossification of spinal ligaments and enthesis. It is a non-inflammatory condition, unknown etiology,
common in elderly. Patients usually complain of stiffness and decreased ROM at the level of the T spine.
 Ankylosing spondylitis demonstrates vertical bridging syndesmophytes rather than flowing osteophytes in DISH.
XRay shows erosive changes in sacroiliac joints including sclerosis, widening, narrowing, partial ankylosis. Usually
dx in 20-30 yo. Anterior uveitis (inflammation of uvea): red eye, pain, photophobia, blurry vision, circumferential
redness (ciliary flush) at the corneal limbus (junction of cornea and sclera). +Schober test: < 4cm in L spine ROM.
 Ankylosing spondylitis: inflammatory back pain/stiffness in spine; worse after immobility, better with use. Prominent
morning symptoms (>1 hour), buttock pain 2/2 BL sacroiliitis. Primarily axial involvement. Usually <45yo, responds
to NSAIDs, +HLA-B27. ASAS (assessment of spondyloarthritis international society) classification criteria. If
negative radiographs: non-radiographic axial spondyloarthritis (bay be prior to bony destruction can be visualized).
Buttock pain 2/2 sacroiliitis; fusion of spine over time, leading to rigidity and kyphosis. Exercise to preserve ROM
and strengthen spine extensor muscles to prevent kyphosis. PT, NSAID first line for symptomatic patients. If patient
does not respond to >2 NSAID types (naproxen, indomethacin) for >4 weeks, then start TNFa inhibitor (per the
ASAS/EULAR guideline-assessment of spondyloarthritis international society/European league against rheumatism).
Response from TNFa inhibitor usually rapid within 6 weeks of therapy.
 DJD/DDD: age related changes in the proteoglycan content in the nucleus pulposus of the vertebral disks, which
causes them to shrink as they become desiccated and more friable. With age, the annulus fibrosus becomes more
fibrotic, less elastic, and shifts in position. Vertebral body endplates adjacent to the disk develop sclerosis and
osteophyte formation at the vertebral margins.
 HBV associated arthritis: rapid onset symmetric bilateral polyarthritis, rash, elevated LFT. Prodrome phase of acute
HBV infection. Test with Hep B core IgM, Hep B surface antigen.
 Joint tap in acute inflammatory monoarthritis to rule out infection. Gout can cause fever and inflammation, but
negative gram stain and +crystals do not rule out infection. Start empiric Abx for suspected infection. Gout is seen
with intracellular crystals. Extracellular PMN in the joint >50K is suggestive of infection. Cover MRSA and GNR and
gonococcal.
 Lyme arthritis: large effusion and stiff joint without much pain. Knee is most commonly affected. Usually
monoarticular/oligoarticular. Diagnose with ELISA screening test for Borrelia burgdorferi, with confirmatory western
blot. Synovial fluid cultures usually negative, showing inflammatory cells with neutrophil predominance. Joint fluid
may be +PCR for B. burgdorferi, but usually rely on serologic tests first, to show immunologic response to diagnose.
 Drug induced SLE: procainamide, minocycline, hydralazine, TNF-a inhibitors (infliximab, etanercept), methyldopa,
diltiazem, INH, anticonvulsants, antithyroid agents, and antibiotics. +ANA, negative anti-ds-DNA. +anti-histone Ab
in older meds (may be negative in newer medications). Minocycline and hydral may cause +p-ANCA syndrome with
small/medium vessel vasculitis with organ involvement.
 Inclusion body myositis (IBM): insidious and slowly progressive inflammatory myopathy. >50 yo M. muscle
weakness involves both distal and proximal sites. Can be asymmetric in 15% of patients. Skin sparing. Rarely
associated with fever, rash, or pulmonary symptoms. Usually normal muscle enzymes. No muscle pain/myalgia. EMG
findings: short-duration, small, low-amplitude polyphasic potentials; fibrillation potentials at rest, and bizarre high-
frequency, repetitive discharges. No pain or stiffness in joints.
 ALS: amyotrophic lateral sclerosis: progressive dysfunction of both UMN/LMN pathways; pure motor deficits. UMN
findings: spasticity, hyperreflexia, pathologic reflexes (extensor plantar response). LMN findings: weakness, atrophy,
fasciculations, cramps.
 MG: myasthenia gravis: fluctuating and fatigable muscle weakness that worsens with activity, and improves with rest.
BL asymmetric ptosis worsened by prolonged upward gaze, snarling smile 2/2 facial muscle weaknes, nasal speech
worsened by prolonged speaking, and limb weakness that increases with exercise.
 Stain induced myopathy usually associated with myalgia (muscle pain), tenderness, and cramping within 6 months.
Normal EMG results.
 Bone scintigraphy: used to visualize areas of bone turnover 2/2 osteophyte formation, subchondral sclerosis,
subchondral cyst formation, and bone marrow lesions. Limited anatomic resolution; uses ionizing radiation.
 Treat keratoconjunctivitis sicca (dry eyes in primary Sjogren syndrome): with artificial tears, topical lubrication, or
punctal plugs. Consider cyclosporin gtt if refractory. Sjogren manifests as inflammation of exocrine glands including
salivary, lacrimal, and pancreatic exocrine glands.
 Olopatadine gtts: reduce histamine release from mast cells. Used to treat allergic conjunctivitis.
 Marfan Syndrome: AD mutation in FBN1 gene responsible for producing Fibrillin 1 (structural protein that contains
elastic fibers, found in arterial walls). Tall stature, anterior thoracic deformity, arachnodactyly, spinal curvature, skin
and joint hyperextensibility. Beware of aortic root dilatation with dissection, rupture, or AI. TTE at time of diagnosis
and 6 months later to determine rate of enlargement. Annual TTE if stability is documented, and more frequent
imaging if maximal aortic diameter is >4.5cm. apical lung bullae can form, leading to PTX.
 Hypersensitivity vasculitis: small vessel disease 2/2 inflammatory response from antibody mediated immune complex
deposition in response to medications. Complement system is activated, which triggers neutrophils to accumulate in
the capillaries, arterioles, and post-capillary venules. This process similar to other small vessel vasculitides. Usually
2/2 medications (antibiotics). Tx: stop the drug, and prednisone if severe systemic symptoms.
 Treat mild SLE with Plaquenil (hydroxychloroquine). Steroid sparing protocol. All SLE patients who can tolerate it
should be on Plaquenil (an antimalarial medication, which helps reduce disease activity, improve survival, and reduce
risk of SLE related thrombosis and MI).
 Treat more severe SLE with azathioprine (non responsive to steroids and Plaquenil). Azathioprine has serious
toxicity, and is supplanted by Mycophenolate mofetil-Cellcept (use for severe lupus nephritis or for
arthritis/pericarditis recurrence while on Plaquenil). NSAIDs + colchicine is first line therapy for pericarditis.
 Colchicine MOA: inhibits microtubule polymerization by binding to tubulin; mitotic spindle poisoning agent.
 Hand OA: diagnose clinically, no need for XR. Pain with activity, relieved with rest. Morning stiffness <30 minutes.
Bony hypertrophy, Heberden, Bouchard nodes palpated. Squaring/boxing of CMC joint at base of thumb. History and
exam is more sensitive and specific than radiography.
 PMR: polymyalgia rheumatica: tx with low dose steroids: prednisone 15mg/day chronically; may taper over 6
months, but may require up to 3 years of steroids. Shoulder and hip girdle pain in elderly. Also with signs of systemic
inflammation: low grade fever, weight loss, malaise, and elevated ESR.
 If GCA: give high dose steroids 60mg/d, with ASA 81 to reduce ocular complications.
 High dose ASA 650 tid functions similarly to other NSAIDS: analgesic, anti-inflammatory, and anti-pyretic.
 Reactive arthritis 2/2 chlamydia trachomatis. Dx with PCR DNA amplification urine test for Chlamydia.
Oligoarticular lower extremity dactylitis, keratoderma blennorrhagicum (yellow-brown rash with vesicular and
hyperkeratotic nodules on soles/palms that are indistinguishable from pustular psoriasis).
 RA algorithm: MTXhigher MTX doseprednisone/NSAID for symptomatic reliefTNFa inhibitorRituxan.
 Plaquenil for RA is only indicated in early, mild, and nonerosive disease. It does not retard radiographic progression
of RA, and should only be used on patients whose disease has remained nonerosive for several years
 Rituximab: anti-CD20 B cell depleting monoclonal antibody: moderate to severely active RA in combination w/ MTX
for refractory response to DMARD (TNFa inhibitor therapy). May consider Rituxan if high disease activity and poor
prognostic features despite sequential nonbiologic DMARDs or MTX in combination with other DMARDS.
 Prednisone may help w/ short term relief of RA flare, but tx progression of RA disease with DMARD in known
seropositive erosive disease to halt bony joint destruction.
 Cryoglobulinemic vasculitis in Hep C: low C4, normal C3, +RF, monoclonal paraprotein spike in IgG band (2/2 RF),
palpable purpura, GN (AKI with active urine sediment and cellular casts), mononeuritis, skin infarctions on
fingers/ears. Ear infarction is most consistent with cryoglobulinemia. Dx with serum cryoglobulin level.
 Milwaukee shoulder syndrome: basic calcium phosphate deposition: pain, stiffness, swelling, progressive over time
s/p trauma or hx of overuse (months later). Old (>70 yo) F with large unilateral joint effusion, with synovial fluid that
is blood tinged with low leukocyte count. Crystals not visible on routine light microscopy, not birefringent (not
appreciated with polarizing microscopy); but are revealed with alizarin red staining (red, globular crystal clumps). XR
shows narrowing of glenohumeral joint, calcification of periarticular cartilage, and erosive changes on humeral head.
Upward subluxation of humeral head 2/2 rotator cuff destruction and bony cysts are common.
 Inflammatory arthritis: septic, crystal (gout, pseudogout-CPPD).
 Diagnose disseminated gonococcal infection (DGI) with cervical/rectal/oral/penile/blood cultures. DGI occurs in 3%
of patients with Neisseria gonorrhea. Can cause bacteremia with vesiculopustular/hemorrhagic macular skin lesions,
fever, polyarthralgia, and tenosynovitis with nonpurulent arthritis and culture negative synovial fluid.
 Patients can also acquire purulent arthritis without rash or features of disseminated GC infection (separate entity from
DGI). In these subset of patients, synovial fluid is often positive for Neisseria gonorrhea.
 Hep B: viral infection, may present with arthralgia, fever, and rash.
 DC MTX if pt plans to get pregnant. MTX is teratogenic and abortifacient. Stop >3 months prior to conception. Take
folic acid; okay to continue Plaquenil in RA/SLE even during pregnancy. Avoid low dose steroids if possible before
14 weeks of gestation (risk of cleft palate).
 IgG4 related disease: lymphoplasmacytic infiltration, and enlargement of various structures (pancreas, lymph nodes,
salivary/lacrimal glands, periaortic region leading to retroperitoneal fibrosis, kidneys, and skin). Most occur in >50 yo
M. presents as subacute development of mass in affected organ, and 90% have multi-organ involvement. Biopsy
reveals IgG4 producing plasma cells, lymphoplasmacytic infiltrate with storiform fibrosis and obliterative phlebitis,
rare neutrophils, and no granulomas. Usually end result is progressive organ fibrosis after enlargement occurs.
 HL: Hodgkin lymphoma: palpable lymphadenopathy or mediastinal mass; biopsy shows clonal malignant Hodgkin
Reed-Sternberg cells (large cells with pale large nuclei containing large purple nucleoli) in background of
granulocytes, plasma cells, and lymphocytes. “starry sky appearance”.
 Sarcoidosis: noncaseating granulomatous multisystem disease; commonly affects the lungs.
 Erosive OA is hard to distinguish from RA (both F>M, polyarticular, inflammatory-red/warm/swollen/tender/stiff,
and preferentially affects hand joints). Erosive OA is more common in DIP and CMC joints, and generally spares
wrists and elbows. Negative inflammatory markers: RF, anti-CCP, ESR/CRP. In erosive OA, lesions are centrally
located in the joint with proliferative changes. In RA, erosions are at joint margins with periarticular osteopenia.
 Limited cutaneous systemic sclerosis (LcSSc): form of systemic sclerosis characterized by distal (face, neck, hands)
skin thickening, spares proximal skin. Not accompanied by internal organ fibrosis. May display CREST features
(calcinosis cutis-calcified nodules on extremities, Raynaud phenomenon, esophageal dysmotility, sclerodactyly,
telangiectasias). PAH is more common in LcSSc than in diffuse cutaneous systemic sclerosis. CREST is variant of
LcSSc characterized by +ANA and +anticentromere Ab.
 Eosinophilia myalgia syndrome: fasciitis and dermal induration 2/2 consumption of contaminated L-tryptophan. Also
with neuropathy, myopathy findings. New cases are rare, since the toxic was identified and no longer used.
 Morphea: localized area of skin thickening usually on torso. Painless discolored patches.
 PBC: chronic cholestatic liver disease, unknown etiology, middle aged women. Fatigue, dry eyes, dry mouth, pruritis.
Jaundice, cutaneous hyperpigmentation, hepatosplenomegaly and xanthelasmas are rarely observed at diagnosis.
 MCTD: mixed connective tissue disease: anti-U1-RNP antibodies (ribonucleoprotein). Overall syndrome that includes
SLE, systemic sclerosis, polymyositis in setting of +anti-U1-RNP Ab. Must have 3/5 features: Raynaud, edema of
hands, sclerodactyly, synovitis, myositis. Usually +ANA, negative anti-dsDNA ab, normal complement, and low
incidence of kidney disease. more likely than SLE patients to have myositis and PAH.
 UCTD: undifferentiated connective tissue disease: not an overlap syndrome. Refers to nonspecific clinical features,
without disease specific findings, and nonspecific positive autoantibodies (ANA). May be a precursor to AI disease.
 BAL w/ biopsy is most appropriate diagnostic test for DAH 2/2 SLE. Triad of hypoxemia, new pulmonary infiltrates
on CXR, and decreasing hematocrit is suggestive of DAH. DAH usually occurs in setting of active SLE flare and 90%
have nephritis. CXR shows bilateral infiltrates sparing lung apices. Lung biopsy shows pulmonary capillaritis with IC
deposition. Will usually require mechanical ventilation and aggressive immunosuppression; mortality up to 70%.
 Lofgren syndrome: self-limiting subset of sarcoidosis characterized by triad of acute BL symmetric nondestructive
periarthritis (of the soft tissues, entheses, tenosynovium around joints), BL hilar lymphadenopathy, and erythema
nodosum. Ankle involvement is classic, but knee, wrist, and elbows may be involved. 90% remit within 12 months.
Clinical triad has 95% diagnostic specificity; further diagnostic tests (radiograph, serologic) are unnecessary. Tx with
NSAIDs (naproxen), colchicine, or low dose steroids.
 Erythema nodosum: associated with infections, medications, systemic disease, and idiopathic. Tender, subQ nodules.
 SLE with hip pain on chronic steroids: acute osteonecrosis. Get MRI to evaluate/diagnose for the early bone edema
when plain films are normal. MRI also prognostic: if >20% of femoral volume shows edema and necrosis, progressive
disease (subchondral fx and femoral head collapse) is the rule, where as smaller infarcts rarely progress. Up to 37% of
SLE patients have osteonecrosis by serial MRI monitoring, but <10% become symptomatic. Cushingoid features
indicate higher risk of osteonecrosis 2/2 enlargement of fat cells in long bones. Increased adipose volume causes
compression of small sinusoidal vessels which lead to interosseous HTN and impairment of arterial flow. Early
radiographic findings: bone density changes, sclerosis, cyst formation subchondral radiolucency producing a
“Crescent sign” indicating subchondral collapse. End stage disease characterized by collapse of femoral head, joint
space narrowing, and degenerative changes. Prednisone > 20mg/d for >4 weeks is risk factors.
 Dermatomyositis: higher risk of malignancy, especially in first 2 years after diagnosis. Higher incidence of solid
cancer such as adenocarcinoma of lung, cervix, ovaries, pancreas, colorectal, stomach, bladder, and non-Hodgkin’s
lymphoma. Risk of ovarian cancer incidence ratio was 10.5, highest among all cancers diagnosed.
 PAN: polyarteritis nodosa: most common medium sized vasculitis that affects mesenteric and renal arteries. Presents
with nonspecific inflammatory symptoms, chronic/intermittent abdominal ischemic pain, neurologic involvement-
mononeuritis multiplex, livedo reticularis, purpura, painful subcutaneous nodules, renal impairment 2/2 diminished
perfusion and flow rather than GN. HBV infection is strong risk factor for PAN. Generally spares respiratory tract.
May also have testicular involvement. Bx of artery shows necrotizing arteritis with medium sized arterial aneurysm
and stenosis on imaging studies (angiography, CTA).
 Goodpasture syndrome: affects kidneys and lungs.
 HSP: immune complex small vessel disease with kidney/neuro/skin involvement and abdominal pain. C4, IgA.
 Meniscectomy and prior injury of knee results in 132x fold increase in rate of TKA/TKR and OA progression. Due to
this, meniscus repair rather than meniscectomy is recommended in younger patients.
 Prosthetic joint infection may occur early (<3 months), delayed (3-12 months), or late (>12 months) post surgery.
Early onset infection presents with effusion, erythema, drainage, and fever. Delayed infection are usually more
insidious with prolonged pain without fever. Late infection presents similarly to acute pain in setting of nidus for
seeding such as vascular catheter or other remote site of infection with hematogenous spread to joint. No need for
imaging studies; workup with synovial and blood cultures, and antibiotic coverage.
 Lifestyle modification for gout: add low-fat dairy products, which decreases risk of flares through uricosuric and anti-
inflammatory properties. Avoid high-fructose drinks (metabolization of fructose leads to higher uric acid generation).
Leafy veggies are high in purine (nucleic acid component which is metabolized to uric acid). Alcohol, shellfish, meat
all trigger gout flares though increased uric acid production (high purine load) and impaired renal urate handling.
 Fibromyalgia: chronic widespread pain, tenderness of skin and muscles to pressure, fatigue, sleep disturbances,
exercise intolerance. Start them on aerobic exercise program, and consider CBT or medications if refractory.
 NSAID risk: GI toxicity/bleeding, cardiovascular disease, HTN, kidney disease. ACR recommends topical if >75 yo.
 Topical diclofenac NSAID is superior to placebo for OA.
 Seronegative RA: negative RF and anti-CCP. Radiographs show erosive disease with symmetric joint space
narrowing, marginal erosion, and mild periarticular osteopenia. Absence of bony sclerosis or osteophytes (OA). Even
though initially seronegative, may later become RF positive. M>W.
 Adult onset still disease (AOSD): multisystem inflammatory disease, high spiking fevers, arthritis, high neutrophil
count, markedly high serum ferritin, nonpruritic salmon colored macular/maculopapular rash on trunk or extremities.
High ferritin is a marker of macrophage activation. ESR/CRP elevated. Diagnosis after excluding infection,
malignancy, and other rheumatologic disease. Yamaguchi classification criteria to diagnose AOSD: major (fever,
rash, joint involvement), minor (splenomegaly, lymphadenopathy, elevated LFT).
 AML: acute myeloid leukemia. Malignancy of myeloid progenitor cells. Bone marrow failure: fatigue, dyspnea, easy
bleeding, fever, high circulating myeloblasts.
 Behcet syndrome: vasculitis that affects small-large arteries, also affects veins. Increased prevalence in Mediterranean
and Asians (east Asia to turkey). Recurrent painful oral ulcers + recurrent genital ulcers, eye involvement, skin
involvement (acneiform lesions), and pathergy (development of pustule following needle stick). Oral ulcers
spontaneously resolve after 1-3 weeks. Eye involvement may be severe (posterior uveitis, retinal vasculitis) and lead
to blindness. Hypopyon (suppurative fluid in anterior chamber) is distinctive. May also have GI ulceration.
 CMV: mononucleosis like illness, fevers, lymphadenopathy, colitis, hepatitis, retinitis.
 OA: unilateral, worse with activity, unicompartmental joint space narrowing, osteophytosis, absence of extensive
inflammation. Start with Tylenol for pain up to 3-4 g/d. Can also start with NSAID as first line (interchangeable).
New research shows that Tylenol does not help pain, and that you should start with NSAID.
 Selective COX2 inhibitor and traditional nonselective NSAID (ibuprofen): both help with inflammation. Selective
COX2 inhibitor (celecoxib) has better GI profile; both promote HTN and may exacerbate kidney disease.
 Primary Raynaud phenomenon: common in 30% of young white women. May be initial symptom of an underlying
fibrosing CTD such as MCTD/systemic sclerosis. Predictive features: severe/prolonged episodes of vasospasm,
asymmetric digital involvement, and abnormal nailfold capillary exam or digital pitting. If persistently symptomatic,
consider peripheral CCB such as nifedipine or amlodipine. Sildenafil and endothelin 1 inhibitors if refractory.
 Digital arteriography: invasive test, normal in Raynaud phenomenon. Use for diagnosis of thromboangiitis obliterans
(“Buerger disease”), a nonatherosclerotic vascular inflammatory disease affecting small/medium vessels, usually seen
in male smokers.
 Primary angiitis of CNS (PACNS): recurrent headache, rapidly progressive encephalopathy without evidence of CVA
or infection. Negative MRI/MRA, LP nonspecific for mild inflammatory process. Next step intracerebral angiography
and brain biopsy. Rare and challenging diagnosis, but treatable with cyclophosphamide and high dose steroids.
Differential dx: reversible vasoconstriction syndrome, infection, intravascular malignancy. MRI/MRA is not sensitive
enough to detect a vasculitis.
 Leflunomide may cause acute hepatitis with RUQ pain and LFT derangements. Occurs in 20% of patients. Usually
reversible if <3x upper limit of normal. If >3x elevated, treat with cholestyramine to decrease the drug levels, as
leflunomide undergoes enterohepatic circulation. Monitor q8-12 weeks while on leflunomide.
 Febuxostat is contraindicated in gout if patient is on azathioprine. Febuxostat: non-purine, non-competitive xanthine
oxidase inhibitor that blocks urate synthesis by inhibiting xanthine oxidase, the final enzyme of urate synthesis from
purine precursors. Azathioprine: purine analogue used to treat IBD, undergoes metabolism via xanthine oxidase.
Febuxostat leads to dangerously high levels of azathioprine. Allopurinol also increases azathioprine levels and is
relatively contraindicated.
 Diltiazem is contraindicated in colchicine; dilt is CYP34A inhibitor; coadministration causes elevated colchicine.
 Takayasu Arteritis: arterial compromise in setting of systemic febrile illness. Midabdominal bruit, leg symptoms,
HTN 2/2 aortic/renal artery obstruction. Young Asian female. Diagnose with aortic arteriography. CT angio or MR
angio may also be ordered instead. Large sized arterial involvement.
 Antiphospholipid antibody syndrome: thrombotic disease occluding arm/aorta/renal arteries. Lupus anticoagulant
causes abnormal prothrombin and PTT. Thrombosis causes elevation in D dimer (fibrin degradation products).
 Mycophenolate inhibits purine pathway in nucleotide synthesis; is at least effective as cyclophosphamide for SLE
including lupus nephritis with fewer and milder side effects.
 MTB of hip joint usually presents as indolent disease process (hip, knee spine-pott disease). constitutional symptoms
are frequently absent, and imaging reveals nonspecific erosions (may be confused with OA). Joint aspiration and cell
count, fluid studies/cultures for MTP. Usually 2/2 reactivation of LTBI (if using TNFa inhibitor).
 Mononeuritis multiplex in SLE: Characterized by abnormal findings in territory of >2 nerves in separate body parts;
foot drop with normal reflexes = injury to peroneal nerve; wrist drop = radial nerve injury. Usually nerve damage
results from vascular disorder affecting vasa vasorum or nerve vascular supply. Diagnose with EMG and NCS
(electromyography and nerve conduction studies), which would show a peripheral neuropathy. Peroneal nerve is most
commonly affected site. RF for SLE associated PN: moderate-severe disease, other neuropsychiatric manifestations.
 Plaquenil may cause neuromyopathy with proximal weakness and areflexia. Biopsy shows vacuoles in muscle cells.
 Transverse myelitis: rapidly progressive paraparesis associated with a sensory level. Autonomic symptoms including
increased urinary urgency, bowel/bladder incontinence, sexual dysfunction usually present.
 The development of explosive onset or severe flare of psoriatic arthritis should raise suspicion for concomitant HIV
infection, which may trigger onset or exacerbate preexisting PA and psoriasis. Skin/joint symptoms usually severe.
 Early localized Lyme disease starts with erythema migrans (70-80% of patients with confirmed infection). Early
disseminated Lyme develops several weeks after initial infection, which manifests as febrile illness with myalgia,
headache, fatigue, and lymphadenopathy.
 Strep pharyngitis is a common trigger of guttate psoriasis (droplet), especially in kids. It may also be the first sign of a
flare in previously stable chronic plaque psoriasis.
 Parvovirus B19 infection: slapped cheek: Treat joint pain with NSAIDs. Kids: polyarthritis with symmetric swelling
and stiffness, flu like symptoms. Serum IgM antibodies to parvovirus B19. Self limited condition; supportive care.
 Wegener granulomatosis with polyangiitis: Systemic necrotizing vasculitis, affecting upper and lower respiratory tract
and kidneys. >70% have airway manifestations (sinusitis, nasal/inner ear/laryngotracheal inflammation). Puprua,
ulcers, common skin manifestations. Diminished hearing, myalgia, arthralgia, epistaxis, mononeuritis multiplex
(weakness on grasp strength), BL conjunctivitis, BL maxillary sinus tenderness to palpation, diffuse rhonchi on exam,
palpable purpura in BL LE, hemoptysis, cough, pleurisy, multifocal infiltrates/nodules on CXR, cavitary lesions;
diffuse opacities if DAH. Pauci-immune GN in 80%. Dx with lung or kidney biopsies. PR3 antibodies diagnostic.
 Antiproteinase 3 (PR3) antibodies: diagnostic of Wegeners.
 Antimyeloperoxidase antibodies: ANCA in perinuclear rather than cytosolic pattern. Associated with microscopic
polyangiitis, eosinophilic granulomatosis with polyangiitis, and rapidly progressive GN (RPGN).
 Cryoglobulinemia: kidney, skin, nerves, and lung involvement. Low complement levels (C3 and C4).
 Confirm lupus nephritis with biopsy, especially if anti ds-DNA antibody titers, complements, and suspicion of GN.
Nephritis occurs in 70% of SLE. Lupus nephritis: proteinuria >500mg/24h, cellular casts (RBC or WBC) in urine
sediment. Anything class 3-5 will need aggressive immunosuppressive therapy (Cytoxan or cellcept). Class 3-5
usually are progressive and proliferative, leads to ESRD within 5-10 years of onset in 50-70% of patients.
 Best BP med for gout patients with hyperuricemia is an ARB (losartan). HTN common comorbidity (in 75% of gout
patients). BP meds have variable effects on serum urate levels. Losartan (not other ARBs) and CCB were associated
with a reduced risk of gout (RR for losartan 0.81 (95% CI, 0.7 - 0.84), (RR for CCB 0.87 (95% CI 0.82 - 0.93). Both
losartan and CCB lowers serum urate; losartan also promotes kidney rate excretion via urate reabsorption transporter.
 In the same study from above, ACE, non-losartan ARBs, BB, and diuretics were all associated with an increased risk
of gout; absolute risk of gout was greatest with diuretics, estimated 6 events per 1000 person-years.
 Pegloticase: refractory persistent hyperuricemia and severe tophaceous gout not responsive to oral urate-lowering
therapy including xanthine oxidase inhibitors allopurinol and febuxostat. Pegloticase is a synthetic uricase
replacement, IV med for treatment-failure gout. Must d/c the XO inhibitors prior to started pegloticase because of
immunogenicity to pegloticase that can result in an infusion reaction; loss of response to the med suggests
neutralizing antibodies and indicates increased risk of infusion reaction, which clinically presents as high serum urate
level. Stop XO inhibitors because they may mask a rising serum urate level that is actually 2/2 pegloticase antibodies
and risk of serious infusion reaction. Pegloticase is contraindicated in G6PD deficiency patients
 Anakinra: interleukin 1b inhibitor: off label use for gout ppx and flare treatments refractory to conventional approach.
 Anti-ds DNA antibody correlates with lupus flare, and specifically for lupus nephritis. May track titer levels.
 ANA is screening tool for SLE (>95% of SLE are positive for ANA); ANA does not have good specificity or
correlate with disease activity. Anti-smith Ab are specific to SLE.
 Treat scleroderma renal crisis (SRC) in setting of diffuse cutaneous systemic sclerosis (DcSSc) with ACEi captopril.
SRC occurs in 10-15% of systemic sclerosis patients; more frequent in DcSSc compared to limited cutaneous
systemic sclerosis. Vascular involvement of afferent arterioles lead to glomerular ischemic and hyperreninemia.
Pathophysiology: noninflammatory vascular process mediated through RAAS system/axis. They typically present
with acute oliguric renal failure, severe HTN, mild proteinuria (nephrotic range proteinuria), UA with few cells/casts,
MAHA, and thrombocytopenia; some develop pulmonary edema, hypertensive encephalopathy (with altered mental
status). Normal BP in 10% of cases. ACEi not only helps with BP, but restores renal function. Captopril is the
preferred choice; most extensively studied and short half-life for rapid titration.
 Treat PAH or finger ulcerations in scleroderma with sildenafil.
 OA: Tylenol <3g/dayNSAID (diclofenac), risk of PUD and GIB and CVDnarcotics.
 Tramadol: centrally acting synthetic opioid analgesic; binds to mu-opioid receptors; inhibits reuptake of
norepinephrine and serotonin. Tolerance occurs with long term use. SE: headache, dizziness.
 Painless frank hematuria in pt with hx of Wegener’s disease on cyclophosphamide (nonbiologic disease modifying
antirheumatic alkylating agent): Think of bladder cancer. Diagnose with cystoscopy. Both Wegener’s and Cytoxan are
independent risk factors for bladder cancer. PO Cytoxan has higher risk; prolonged daily exposure to the metabolites
which cause mucosal irritation and metaplasia. Incidence of cystitis/bladder CA lower with IV Cytoxan, especially if
given with mesna (adjuvant therapy to detoxify the urotoxic metabolites).
 Systemic sclerosis induced coronary vasospasms and microvascular disease lead to patchy myocardial fibrosis; leads
to cardiomyopathy with heart failure in systemic sclerosis. Accelerated macroscopic coronary atherosclerosis may be
associated with autoimmune inflammatory disease; but the ischemic insult with systemic sclerosis is microvascular,
with ischemia 2/2 structural changes, and recurrent small vessel spasms throughout. This results in contraction band
necrosis (pathologic finding 2/2 episodic myocardial ischemia followed by reperfusion). Ultimately leads to patchy
fibrosis, and cardiomyopathy, with HFrEF. Poor prognosis, mortality of 75% at 5 years. Will present with CHF s/s.
 Constrictive pericarditis: chronic inflammatory disorder resulting in fibrosis of pericardium, loss of elasticity, and
noncompliance of the tissue. May be associated with connective tissue disease, or infection. Pericardial knock, pulsus
paradoxus, TTE with impaired cardiac filling and abnormal pressures.
 Diffuse cutaneous systemic sclerosis may lead to blind loop syndrome (malabsorption 2/2 bacterial overgrowth) 2/2
altered peristalsis from the underlying fibrosis. Diagnose with glucose hydrogen breath test; gold standard is small
bowel aspiration for detection of bacterial overgrowth; usually presents as abdominal cramping, weight loss, loose
stools. Usually no dysphagia, nausea, or vomiting.
 OA with calcium pyrophosphate deposition (CPPD): pyrophosphate arthropathy. Symptoms consistent with a
degenerative process, and signs of OA in hands. XR shows chondrocalcinosis (calcification densities). CPPD usually
involves common OA joints and noncommon non-weight bearing joints (shoulders, wrists).
 Gout RF: postmenopausal, HTN, diuretic, DM2, overweight, pacific islander, shellfish diet, high meat intake.
 The degree of ESR elevation is proportional to the serum globulin concentration, albumin-globulin ratio, serum
fibrinogen concentration, and hemoglobin A1c, but not fasting serum glucose concentrations. The ESR is dictated by
characteristics by the RBC and presence of plasma proteins that alter the normal repulsion forces between RBCs
which affect their ability to aggregate, form rouleaux, and sediment more quickly. Plasma proteins include serum
fibrinogen (acute phase reactant produced by liver in response to Proinflammatory cytokines), infection, malignancies
that neutralize negative surface charges and increase the ESR. Noninflammatory conditions (CKD, DM, pregnancy,
obesity, aging process) may also cause increased fibrinogen which results in higher ESR.
 Pericarditis is the most common cardiac manifestation of RA, and is usually asymptomatic. RA is an independent RF
for CAD and CHF. Severe extra-articular disease is at an increased risk of cardiovascular death. 1/3 of RA has an
asymptomatic pericardial effusion, and 10% will present with symptomatic pericarditis at some point during the
course of their disease. Diagnosed when 2/3 is positive: pleuritic chest pain, friction rub, diffuse STE on EKG.
 Synovial fluid >2000 WBC is inflammatory: gout, pseudogout, septic joint. Presence of crystals do not rule out infx.

MKSAP: PULMONOLOGY & CRITICAL CARE MEDICINE

 Heliox: helium and oxygen mixture (helium is less dense than nitrogen). Use for pts with increased airway resistance.
 Subcentimeter pulmonary nodule: use Fleischner criteria to establish timing of next imaging.
 Evaluate worsening COPD symptoms with repeat spirometry testing. Helps determine declining lung function.
 Roflumilast: PDE-4 inhibitor for severe/refractory COPD with recurrent exacerbations. inflammation. Minimal
bronchodilation effects. Should always be combined with long-acting inhaler.
 Pulmonary lymphocytic carcinomatosis. Lymphangitic spread is usually associated with melanoma, lymphoma,
leukemia, and adenoCA (lung, breast, GI),
 CT findings in IPF (idio pulm fibrosis): BL peripheral and LL predominant interstitial thickening w/ honeycombing.
 Radiation pneumonitis: cough/DOE >6w after radiation exposure. CT shows hazy opacities, and GGO. Usually
resolves in 6 months but may progress to well demarcated localized fibrosis w/ volume loss and bronchiectasis.
 Long term home O2 in COPD patients indicated if: PO2 <55 or SaO2 <88% on RA, e/o PAH (edema 2/2 RHF,
polycythemia, PO2 <60 or PaO2 <88% on RA).
 A low FEV1/FVC ratio is consistent with obstructive ventilatory defect. If <70%, diagnostic. Degree of reduction in
the FEV1 is used to characterize degree of obstruction. FEV1 50-80%: moderately reduced. FEV1 34-50%: severely
reduced. FEV1 <34%: very severely reduced.
 Increased lung volumes and higher than predicted TLC= hyperinflation. High residual volume=air trapping. COPD.
 Lower than predicted DLCO suggests lung parenchyma disease, in COPD usually 2/2 emphysema.
 Bronchiectasis: sputum daily, thickened airways (in contrast to vessels), collapsible and inflammated airways.
 OHS: BMI>30, PCO2 >45. PFT=restrictive w/o obstruction, with FEV1 and FVC but preserved FEV1/FVC ratio.
 For vascular access: flow through catheter is inversely proportional to length, and proportional to radius^4.
 Small cell: 15% of all lung cancers. Smoking. Large hilar mass w/ bulky mediastinal lymphadenopathy. Associated
with LES. SCLC is most common pulmonary neuroendocrine tumor in adults. SCLC is more aggressive than non-
SCLC, usually disseminated on diagnosis. Sensitive to radiation/chemotherapy.
 Criteria for lung transplantation in advanced COPD: hx of XC associated with acute hypercapnia (PCO2>50);
PulmHTN, cor pulmonale, or both despite O2 therapy; FEV1<20% with DLCO<20% with homogenous distribution
of emphysema on CT. Absolute contraindications for transplantation: malignancy within last 2 years, Hep B/C
cirrhosis, active/recent smoking, drug use/alcohol, psychiatric illness, noncompliant with medical care, absent social
support. >65yo is a relative contraindication.
 LVRS: lung volume reduction surgery: upper lobe predominant emphysema. Higher mortality if FEV1<20%.
 HRCT: high resolution CT of lung: for diffuse parenchymal lung disease. Don’t use it to evaluate solitary pulmonary
nodules or masses, as the intervals between imaging slices is usually >1cm.
 Portopulmonary HTN: Pulm HTN in setting of portal HTN 2/2 cirrhosis. Group A PAH. Other causes of Group A
PAH: HIV, connective tissue disease, drug toxicity.
 Hepatopulmonary syndrome: intrapulmonary shunting of blood 2/2 dilated pulmonary vessels. Causes R>L shunt.
 Pulmonary veno occlusive disease (PVOD): rare cause of PAH 2/2 fibrous occlusions of distal pulmonary veins.
Usually diagnosed in young adults/children.
 VAP: 48 hours post intubation. Undergo lower respiratory tract sampling, and f/u on micro for new organisms.
 EPAP component of NIPPV helps maintain airway patency and recruits atelectatic alveoli. It also counters the
increased workload imposed by high airway resistance.
 MCC of bacterial pathogens in COPD XC: H. influenza, H parainfluenzae, strep pneumo, moraxella catarrhalis.
 COPD XC: pred 40 x5d, azithro.
 PAH seen on TTE. Dx with O2 measurement during sleep and exertion prior to RHC, for PAH group 3 (lung disease).
PAH group 3 is 2/2 chronic hypoxia, causing diffuse pulmonic vasoconstriction, resulting in vascular remodeling and
sustained PH. Tx focuses on underlying lung disease and addressing hypoxia. RHC is for unknown etiology w/u.
 Treat moderate persistent asthma with a low dose inhaled steroid and LABA in addition to a SABA. May consider
long acting anticholinergic if the steroid+LABA combo does not improve symptoms. Definition of moderate
persistent is FEV1 between 60-80% which responds to bronchodilator therapy.
 Leukotriene agonist or low dose inhaled steroid are recommended for patients with mild persistent asthma, which is
defined as asthma symptoms >2 days/week, with FEV>80%.
 Bronchial carcinoid tumor: usually involves proximal airways, presents with endobronchial narrowing/obstruction
which leads to postobstructive PNA. Suspect in pts with persistent pulmonary infiltrate and localize wheezing. Most
common lung cancer in kids/adolescents. Only 1-5% have carcinoid syndrome (flushing, diarrhea, abdominal pain,
bronchoconstriction, nausea/vomiting, restrictive cardiomyopathy).
 ABPA: recurrent asthma exacerbations in setting of peripheral eosinophilia, elevated IgE, recurrent fevers,
bronchiectasis, productive cough with thick brown mucus plugs that results in bronchial airway obstruction with
associated atelectasis. Responds to steroids.
 Pleural effusion: transudative vs exudative (light criteria). Exudative effusion: uncomplicated (negative gram stain
and culture, pH >7.2, glucose >60, no loculations) vs complicated (glucose <60, glucose ratio fluid/serum <0.50,
pH<7.2, +cultures, LDH >1K). Empyema: frank pus, >50K WBC in fluid.
 Uncomplicated parapneumonic effusions: no chest tube needed. Abx tx only.
 Complicated paranpneumonic effusion: small bore chest tube and empiric abx to prevent formation of pleural “peels”
that may trap lung and cause loss of lung function. Loculated effusions may required multiple tubes, tPA, or surgical
decortication treatment.
 Malignant effusion: serial taps, pleurodesis, indwelling pleural catheter (pleurex cath).
 Vocal cord dysfunction/paradoxical vocal ford motion disorder: adduction of vocal cords (instead of abduction)
during inspiration; involuntary maneuver, seen in asthmatics. S/S: chest tightness, wheezing, dyspnea, and no
improvement with inhalers. Gold standard for dx: laryngoscopy (ENT), tx with laryngeal control techniques,
biofeedback, relaxation techniques, SLP eval.
 Large pneumothorax (>2cm); treat with needle decompression/aspiration and high flow oxygen supplementation. The
high flow O2 helps facilitate absorption of pleural air.
 If stable large primary spontaneous pneumothorax, treat with needle decompression, and follow up imaging. For
unstable or symptomatic patients with secondary pneumothorax, insert chest tube.
 PAH group 2: due to underlying cardiac disease (diastolic heart failure). Tx afterload reduction, reduce BP.
 Epoprostenol: advanced IV therapy which causes pulmonary vasodilation, used for group 1 PAH.
 Sildenafil: pulmonary vasodilator for group 1 PAH.
 Treat severe persistent asthma with omalizumab, humanized monoclonal IgE antibody, used for asthmatics refractory
to steroids, with e/o allergies to perennial aeroallergens and with serum IgE 30-700. SQ q2-4 weeks.
 Avoid ventilator induced lung injury by reducing TV to 6cc/kg, and peak pressures <30.
 TRALI is the last severe form of ARDS. Risk is highest with large volume, multiparous female donor.
 Consider thoracoscopy for negative cytology on plueral fluid analysis in patients with high suspicion of malignancy.
 Mesothelioma: pleural thickening on imaging studies, rapid accumulation of exudative pleural fluid, pleuritic chest
pain, systemic weight loss, and exposure to asbestos. Pleural fluid yield on cytology for cancer: 65%+25%+5%
 Initial vent settings: continuous mandatory ventilation with volume control.
 Confirm occupational asthma with before and after spirometric testing, after exposure to the environment.
 Diisocyanates from spray painting in auto body shop is associated with triggering bronchospasms.
 Gold standard for OSA is CPAP. May use oral mandibular advancement appliance if noncompliant. They increase
upper airway caliber via advancing the mandible by traction, and prevents posterior displacement of tongue via
suction. Tx OSA by maintaining upper airway patency.
 Acute obstructive PE with hemodynamic collapse: EKG S1Q3T3 pattern 2/2 RV strain, not always seen.
 Lung volume reduction surgery (LVRS) candidacy: severe COPD. Symptomatic despite maximal pharmacologic
therapy. Must have completed pulmonary rehabilitation. E/o BL predominant upper lobe emphysema on CT. post-
bronchodilator ELC >100% and residual lung volume >150% of predicted. Maximum FEV1 >20% and less than 45%
of predicted. DLCO >20% of predicted. PCO2 <60, PO2 >45.
 LVRS results in higher mortality when patients have severe emphysema and FEV1 <20%, and DLCO <20% with
homogeneous emphysema on HRCT scan.
 Vilanterol: new ultralong acting B2 agonist, used qdaily.
 Eval chronic cough with +FHx of asthma with methacholine challenge test, especially if normal spirometry and
clinical symptoms suggestive of bronchospastic disease.
 Cough-variant asthma: reversible airway obstruction, but primary presenting symptom is nonproductive cough.
 Classic presentation of asthma: obstructive pattern, with bronchodilator response (200cc and 12% increase in FEV1).
 Bronchial challenge test: FEV1 after exposure to methacholine (cholinergic that induces bronchospasm).
 IM epi has more rapid and consistent absorption. IV > IM if compromised muscle perfusion 2/2 shock state.
 In patients on BB in anaphylactic shock, give glucagon for hypotension/bradycardia resistant to epi. Inotropic and
chronotropic effects are not mediated through B receptors.
 Beware of second biphasic response in anaphylaxis; steroids (methylprednisolone) as adjunctive therapy for the
delayed or protracted reaction that may occur.
 Hypoxia altitude stimulation test (HAST): use to assess air travel in patients with lung disease who sat 92-95% on
RA. If >95% on RA, then no need for further testing. High risk patients: COPD, PAH, restrictive lung disease.
 If HAST test unavailable, then perform 6 minute walk test. SpO2 <84% suggests significant desaturation with
altitude, and warrants supplemental O2. If, SpO2 >84% on 6 minute walk, pt should be referred for HAST testing.
 Viral infections (rhinovirus) may cause acute asthma exacerbation, but sometimes will require a step-up in therapy.
 Viral induced bronchospasm usually resolve after 6-8 weeks. If persistent, step up from SABA to inhaled steroids.
 RF for OSA: male, snoring, obese.
 RF for CSA (central sleep apnea): AF, CHF.
 Out of center sleep study test: for patients with high pretest probability of uncomplicated moderate-severe OSA
 In-laboratory overnight PSG is more appropriate for patients with: CHF, advanced pulmonary disease, risk of CSA.
In lab PSG allows for more detailed analysis ad intervention; technician may start them on PAP and asses response to
therapy. Tx OSA with CPAP, Tx CSA with ASV (adaptive servoventilation). May also bleed in O2 as needed.
 Silicosis is an independent risk factor for lung CA. Upper lobe predominant disease, with fibrosis/scarring/nodularity.
 Chest MRI is limited in assessing parenchymal lung. Good for evaluating chest wall/pleural/mediastinum pathology.
 Silicosis is also an independent risk factor for pulmonary TB.
 GOLD: global initiative for chronic obstructive lung disease: risk category for COPD patients.
 mMRC (modified medical research council questionnaire) and CAT (COPD assessment test) are used to objectively
quantify symptomatology in COPD patients.
 GOLD A COPD: FEV1/FVC <70%, and FEV1 >50%, 0-1 exacerbations in past year. Infrequent/mild symptoms.
 GOLD A: mMRC 0-1 or CAT <10. Tx GOLD A with SABA prn.
 GOLD B: mMRC >2, CAT>10. SABA prn, LABA vs LABA/LAMA scheduled (long acting muscarinic agent)
 GOLD B have same spirometry measurements and exacerbation frequencies as GOLD A, but are more symptomatic.
 GOLD C: FEV1/FVC <70%, FEV1 <50%. >1-2 exacerbations annually, or >1 hospitalization for COPD XC.
 Tx GOLD C mMRC 1-2, CAT <10 with same as above. Step up therapy includes PDE4 inhibitor (phosphodiesterase)
 GOLD D: same spirometry/exacerbation freq. as GOLD C, but much more symptomatic (mMRC >2, CAT >10).
Treat with SABA prn, inhaled steroids and LABA or LAMA and pulmonary rehabilitation. Alternatively therapy
includes triple combination of LABA/steroids/PDE4 inhibitor, or double combination of 2 LABA.
 In patients with asthma and persistent suboptimal control despite therapy, with GERD symptoms, trial empiric PPI.
 Respiratory symptoms are common in GERD, and reflux presents as chronic cough. GERD is common in asthma, up
to 90% of asthmatics may have some reflux. GERD may exacerbate and make asthma worse. Treating GERD may
improve asthma control and symptoms.
 CSA and HF manifests as Cheyne-stokes breathing pattern. Beware of CSA 2/2 narcotics. CPAP may exacerbate CSA
 ASV: adaptive servoventilation: form of PAP therapy used in CSA; provides variable inspiratory pressure with low-
level expiratory PAP, with baseline respiratory rate. If patients breathe below the set rate, ASV delivers pressures.
 ABPA: chronic hypersensitivity reaction in response to colonization of lower respiratory tract with Aspergillus. Seen
in CF and atopic asthmatic patients. Inflammatory process causes impaired mucociliary clearance, difficult to control
asthma, and destruction of pulmonary parenchyma, with bronchiectasis. Symptoms include severe/uncontrolled
asthma, productive cough, mucus plugs, fevers, and fatigue. Radiograph shows recurrent infiltrates and centrilobular
bronchiectasis. +skin test to mold (aspergillus antigen), peripheral eosinophilia >1K on CBC, IgE >1000. Tx with
standard asthma therapy and lowest dose of steroids to control symptoms. Consider antifungal and omalizumab.
 CF: autosomal recessive, abnormal chloride transporter. Thick viscous secretions in lung, pancreas, liver, intestines,
and reproductive tract, associated with multi-organ dysfunction. Diagnosed in children and adolescents.
 Eosinophilic granulomatosis with polyangiitis: Churg-Strauss syndrome. Autoimmune small vessel vasculitis.
Peripheral eosinophilia and involves lung as asthma. Purpura on hands/legs, sensory/motor neuropathy.
 ProCESS 2014 trial: Q: among pts with early septic shock, is EGDT or novel protocol-based therapy superior to usual
care in reducing all-cause in-hospital mortality at 60 days? Study: 31 ED sites randomized, septic shock patients to 3
groups for 6 hours of resuscitation: (1) protocol based EGDT, (2) protocol tx that did not require CVC/inotropes or
transfusions, (3) usual care. Results showed that protocol based resuscitation did not improve outcomes. CVC is not a
priority, fluid resuscitation is more urgent.
 Median survival of IPF (idiopathic pulmonary fibrosis) is 3-5 years. ATS (American thoracic society) recommends
hospice and palliative care rather than intubation/mechanical ventilation for advanced progressive IPF given
irreversible nature of disease, without long term benefit.
 High AGMA, serum gap: methanol (wood alcohol) vs ethylene glycol (antifreeze) intoxication. Methanol is
metabolized to formic acid, which is retinotoxic. Ethylene glycol is converted by alcohol dehydrogenase to oxalic
acid, which crystallizes in renal tubules and causes nephrotoxicity. Conversion to toxic metabolites and clearance of
alcohol is diminished by ethanol or IV fomepizole. Consider urgent dialysis for removal.
 Isopropyl alcohol causes intoxication with an osmolar gap, but without AGMA.
 Pleural ADA for extrapulmonary TB has Sn 95% and Sp 96% when levels are >60 U/L. High clinical suspicion in
patients with lymphocytic predominant exudative effusion. ADA <40 virtually excludes TB effusion. Tx empirically
for high ADA, continue workup with pleural biopsy. In absence of tx, 6% risk developing pulm/extrapulm TB in 5y.
 Pleural fluid AFB stain positive <5% of cases, and cultures positive in only 20-30% of cases. Poor yield.
 Asbestosis: slowly progressive pulmonary fibrosis, pleural plaques on imaging studies. A type of pneumoconiosis.
Occupational exposures: construction, automotive servicing and shipbuilding industries. Prolonged latency period
(15-35 years) between exposure and development of asbestosis.
 IPF (idiopathic pulmonary fibrosis) is the most common form of IIP (idiopathic interstitial pneumonia). IIP usually
affects patients later in life, associated with dust exposure, and diagnosis of exclusion.
 Prone positioning (>16h/day) is associated with improved survival for patients with severe ARDS. Mechanism by
which prone positioning improves oxygenation: optimization of VQ matching.
 NO in ARDS improves oxygenation by temporarily improve VQ mismatch, but no mortality benefit.
 To diagnose malignant pleural effusion, repeat thora and pleural studies (cytology) to increase the yield (sensitivity).
 Sensitivity yield of pleural fluid cytology is 60%. 65% of positive results are from initial thora, then  +27%  +5%
 Thoracoscopic pleural biopsy is indicated for all undiagnosed exudative pleural effusions s/p two fluid samplings. The
biopsy is >90% sensitive for pleural biopsy.
 Order flow cytometry on pleural fluid for lymphocytic predominant effusions, when lymphoma is on Ddx.
 VQ scan is the most sensitive imaging diagnostic study for CTEPH (chronic thromboembolic PAH), which p/w
progressive exertional dyspnea w/ e/o cor pulmonale, defects on VQ. Loud S2 (pulmonic), clear lungs, hypoxia,
RVSP, RV hypertrophy. If abnormal VQ, RHC w/ pulmonary angiography is required to confirm the diagnosis and
characterize degree of thrombosis. VQ can differentiate CTEPH (group 4 PAH) from PAH (group 1 PAH).
 In operative candidates, CTEPH is managed surgically w/ pulmonary thromboendarterectomy. Pulmonary angio is
performed preoperatively to assess extent/distribution of thrombus, but not appropriate initial test.
 CT angio (CTPE) can identify proximal pulm arterial webs and luminal narrowing, but it is less sensitive than VQ.
 RL intrapulmonary shunt: may be anatomic vs physiologic, but results in hypoxemia difficult to correct with
supplemental O2. Can be 2/2 collapsed RLL, with perfusion of nonventilated alveoli. Most cases of collapsed lung do
not cause such severe shunt, because of hypoxic vasoconstriction in arterioles in lungs that are not ventilated.
 Hepatopulmonary syndrome, intracardiac shunt, pulmonary AVM, intrapulmonary shunt are all anatomic shunts.
 If diagnostically uncertain, echo bubble study shows LA/LV bubbles after several cardiac cycles.
 Hepatopulmonary: cirrhotic patients who develop hypoxemia in the absence of other cause. The shunt worsens with
upright posture (orthodeoxia).
 In PAH group 1 with negative vasoreactivity test, CCB are not beneficial. PDE-5 inhibitors (sildenafil, tadalafil) or
endothelin receptor antagonists (bosentan, ambrisentan) are preferred. Prostanoids are available only as parenteral
(epoprostenol) or inhaled (iloprost) forms and reserved for advanced/severe disease with rapid progression despite
oral therapy. Additional treatment includes supplemental O2 and chronic warfarin. Women are counseled to avoid
pregnancy. In patients with positive response to vasoreactivity test (decrease mean PA pressure >10 without reduction
in CO or BP), PO CCB (nifedipine) is a reasonable initial therapy. Supervised exercise improves QOL/tolerance.
 Criteria for successful SBT (spontaneous breathing trial) include: tolerate weaning trial >30 minutes, RR<35, SaO2
>90% without arrhythmias, sudden increase in HR/BP or development of respiratory distress, diaphoresis, anxiety.
 SBP: T piece trial (only O2 delivered, no positive pressure), or adjusting vent to apply enough pressure to overcome
resistance of the ETT.
 Direct extubation to NIPPV is effective at weaning pts with obstructive lung disease, but not necessary for primary
hypoxemic respiratory failure 2/2 PNA.
 Workup for hypoxemia: CXR, PFT, CTPE, VQ, TTE, RHC.
 Limited cutaneous systemic sclerosis: associated with pulmonary vascular disease (direct proliferative effects on
vascular walls that obliterate pulmonary arterioles and capillaries, leading to fibrosis of lung interstitium), which lead
to PAH group 1.
 TTE may underestimate true PA pressure; patients need RHC for confirmatory pulmonary hemodynamics, and
determine if there is specific response to vasodilator infusion, which can help guide therapy. RHC with mean PA
pressure >25, normal PCWP is diagnostic of PAH.
 Pleural effusion: dullness to percussion, no egophony, diminished breath sounds, decreased tactile fremitus.
 Malignant mesothelioma arises from pleural cells, and causes slow constant gradual progressive pleural effusion.
 Smoking is not direct risk factor for mesothelioma, but because both increase lung cancer, smoking should be stopped
 Consolidation: increased tactile fremitus, +egophony.
 Diffuse systemic sclerosis associated parenchymal lung disease: w/u with PFT, anti-SCL 70 antibody titers, HRCT
scan (high resolution CT). suspect when skin thickening in distal extremities, which suggests diffuse cutaneous
systemic sclerosis (dcSSc). dcSSC are more likely to develop diffuse parenchymal lung disease (DPLD), serositis, and
renal disease (scleroderma renal crisis).
 DPLD commonly occurs in pts with dcSSC and elevated Ab titers to anti-SCL 70. Alveolitis may precede DPLD.
Presents with dyspnea, nonproductive cough, exercise intolerance. Findings: fine bibasilar Velcro-like inspiratory
crackles, restrictive impairment (decreased lung volumes, decreased DLCO). HRCT shows alveolitis, reticular linear
opacities.
 Anti-PM Scl Ab is associated with myositis. Anti-centromere Ab is associated with limited cutaneous systemic
sclerosis, with high risk of developing PAH. Anti-neutrophil cytoplasmic antibody is associated with granulomatosis
with polyangiitis (GPA-wegener’s polyangiitis), microscopic polyangiitis, and eosinophilic granulomatosis with
polyangiitis (churg-strauss syndrome).
 Long term steroids: hyperglycemia/DM, muscle weakness/catabolism, osteoporosis, AMS/psychosis, decreased
functional status, lipodystrophy, etc.
 Some studies show that macrolide Abx can reduce exacerbation in subset of COPD patients. Can help with CF to
improve lung function. However, risk of increased bacterial resistance, esp. in MAI infection; also QTc prolongation.
 Tx hyperthermic non-exertional heat stroke with evaporative cooling (spray water and cool with fan/convection). Risk
factors: alcohol, age, chronic co-morbidities, and meds (anticholinergic, sympathomimetic, diuretics).
 Causes of hyperthermia: non-exertional heat stroke, malignant hyperthermia, NMS (neuroleptic malignant syndrome)
 Ice water immersion is for patients with exertional heat stroke; usually younger, needs less monitoring.
 NMS: rhabdo (muscle lead pipe rigidity), AMS, hyperthermia, hemodynamic dysregulation hyporeflexia. Tx with
dantrolene, bromocriptine. Look for medication exposure (antipsychotics).
 Serotonin syndrome: treat with cyproheptadine. Characteristic muscle rigidity, AMS, tremor, hyperreflexia, clonus.
 Clinical indications for need of ventilatory assistance (NIPPV, Mechanical ventilation): FVC<50%predicted,
VC<60% predicted (<1L), MIP (maximum inspiratory pressure) < -30 cc H2O. Trial nocturnal intermittent NPPV,
and if progression, may need trach/vent. Especially for neuromuscular weakness (dystrophy, ALS, etc).
 TCA OD: give NaHCO3. Somnolence, hypotension, wide QRS (>100ms), seizure, anticholinergic signs (fever,
tachycardia, mydriasis, reduced bowel sounds). NaHCO3 increases serum pH, which inhibits ionized form of the drug
that binds sodium channels, making it less available. It also increases extracellular sodium, increasing the
electrochemical gradient across myocardial cell membranes and minimizes effect of blockade on sodium channels.
NaHCO3 improves myocardial contractility and reduces risk of arrhythmias.
 TCA: inhibits fast sodium channels in myocardium and conduction system. Causes decreased ventricular function and
increased duration of repolarization that predisposes ventricular arrhythmias.
 Calcium gluconate: Tx for CCB OD. CCB OD: hypotension, encephalopathy. Verapamil and diltiazem (NDHP) is
associated with bradycardia, but this is not usually seen with dihydropyridine such as amlodipine.
 Fomepizole: alcohol dehydrogenase inhibitor. Prevents breakdown of methanol/ethylene glycol into toxic metabolites.
 Physostigmine: antidote for anticholinergic toxicity. May precipitate asystole in setting of TCA OD.
 Stable subcentimeter pulmonary nodule after 24 months of imaging surveillance does not need further evaluation or
imaging if low risk features/characteristics.
 Fleischner criteria: repeat chest CT for pulmonary nodules (subcentimeter) if high risk patient: smoking, other RF.
 Criteria for imaging if subcentimeter: f/u CT 6-12 months, and then 18-24 months if no change. Characteristics of
malignancy on imaging studies: GGO, partly solid nodules,
 Recommendations: annual low dose CT chest in 55-80 yo patients with 30 pack year smoking hx, who are currently
smoking, or have quit in most recent 15 years
 Features for early readmissions for COPD XC: male, age, inadequate TOC Rx, <2 days or >5 days of hospitalization,
and comorbidities (CHF, lung CA, anxiety, depression, osteoporosis). Rate of early readmission for COPD XC is
directly proportional to the number of comorbidities.
 Bronchiectasis: chronic cough, purulent sputum, recurrent pneumonia, PFT with mild/moderate obstructive process,
hx of pertussis as child with subsequent lung damage, CT shows linear atelectasis, dilated and thickened airways with
tram/parallel lines, airway dilatation with lack of tapering, bronchial wall thickening out of proportion to adjacent
vessel, and cysts. After diagnosing bronchiectasis, search underlying cause: CF, Ig deficiency, mycobacterial disease.
 Auto-PEEP: auto-positive end expiratory pressure in severe COPD. Occurs in obstructive lung disease, which slows
expiratory flow rate; does not allow for complete exhalation prior to next breath initiated by ventilator (breath
stacking). The volume that remains in the lung builds up with each breath, resulting in increased intrathoracic pressure
which progresses to hypoxemia, cardiovascular collapse (VR/CO), and barotrauma. Tx by disconnecting the
patient’s ett form the ventilator circuit to allow for longer exhalation, which enables trapped intrathoracic air to
escape, and the intrathoracic pressures to drop, for venous return to improve. You can either RR, or TV, or change
the I:E time to increase exhaled volume with each cycle.
 Partial ETT obstruction (kink, obstructing mucus plug) causes rise in PIP (affects resistance to flow of air through the
ETT), but normal plateau pressures (unaffected by changes in resistance, since it is measured in absence of flow).
 If pt has anxiety and hyperventilates/over breathes the vent, sedation may slow down RR to avoid auto-PEEP.
 For advanced, end stage COPD, hospice is appropriate if FEV1<30%, O2 dependence, multiple hospitalizations for
COPD XC, significant comorbidities, weight loss/cachexia, decreased functional status, and evidence of multi-organ
failure, with cor pulmonale, etc.
 Tx acute aortic dissection with IV esmolol gtt (decreases BP and HR, reducing shear stress on wall). Esmolol is very
short acting. NDHP CCB (verapamil/diltiazem) are second line agents. Target SBP 100-120, HR<65.
 Fenoldopam gtt: dopamine agonist that is anti-HTN. Lowers BP without impairing renal perfusion. Use if HTN
Emergency with AKI. Does not reliably lower HR (okay to use if brady).
 Hydralazine gtt: direct vasodilators. Increases aortic wall shear stress. Difficult to use, unpredictable BP control.
 Nitroglycerin gtt: venodilation>arteriolar dilation. Nitro does not have chronotropic effects (no effect on HR).
 CF in adults: chronic asthma like symptoms, chronic sinusitis, nasal polyposis, recurrent pancreatitis, male infertility,
nontuberculous mycobacterial infection, ABPA, bronchiectasis, positive sputum culture for Burkholderia cepacia or
pseudomonas aeruginosa, hyperinflation on CXR, obstructive ventilatory defect on PFT. Diagnosis is made by
biochemical (sweat chloride, nasal potential difference) or genetic (CFTR mutation) tests.
 Alpha 1 antitrypsin: inhibitor of proteolytic enzymes; a1 antitrypsin deficiency leads to accelerated emphysema and
liver disease from proteolysis. CXR reveals bullous changes prominent at bases.
 ANCA Ab useful in diagnosing GPA (Wegener’s disease), which presents as upper and lower airway disease, renal
involvement, pulmonary disease (diffuse opacities, nodular lung disease, transient pulmonary infiltrates and hilar
lymphadenopathy). Affects older patients, more acute presentation; ANCA Ab also associated with eosinophilic
granulomatosis with polyangiitis (Churg Strauss syndrome).
 Bronchoscopy is indicated for evaluation of new respiratory symptoms that may be 2/2 airway pathology (hemoptysis
or stridor). Lung cancer in high-risk patient may present as hemoptysis 2/2 endobronchial lesion.
 Massive hemoptysis is >100cc of frank blood. Respiratory infectious can also rarely lead to hemoptysis; unusual for
pulmonary parenchymal infections to cause massive hemoptysis.
 Radiation pneumonitis: p/w cough/dyspnea 6-12 weeks after exposure. RF: high total dose of radiation, pre-existing
lung disease (especially COPD), and concomitant chemo/radiation. May be severe, progressive over days - weeks
leading to acute respiratory failure. Gemcitabine is associated with both radiation pneumonitis and drug-induced
pneumonitis. Pathognomic finding on CT: nonanatomic straight line demarcating involved vs uninvolved lung
parenchyma w/ GGO. Usually resolves in 6 months, but may progress to well-demarcated area of fibrosis, with
volume loss and bronchiectasis.
 Radiation fibrosis: may occur in patients without hx of radiation pneumonitis. 6-24 months after exposure, represents
long-term fibrotic sequelae of lung damage. Worsening dyspnea with marginal lung function, irreversible.
 Severe hypothermia with hemodynamic instability should undergo extracorporeal rewarming including
cardiopulmonary bypass to maximize rate of temperature correction and provide hemodynamic support (can increase
body temperature by 9˚C per hour). Passive rewarming (blankets, insulation), and active rewarming (heating pads,
radiant heat) are for moderate hypothermia without hemodynamic instability.
 Other methods of active internal warming (IV warm fluids, intraperitoneal/pleural irrigation) are indicated for severe
hypothermia but with hemodynamic stability. 1L of NS at 42˚ will raise patient’s core by 0.25˚C
 Hypothermia causes ventricular tachycardia and fibrillation, and data suggests that core temperatures >34˚ are needed
for lidocaine to be effective. Electric cardioversion is ineffective if core temp is 30˚.
 Severe anoxic encephalopathy in setting of hypothermia portends a good prognosis with good neurologic outcomes;
the neurologic assessment/exam is unreliable in patients with severe hypothermia. Given potential for recovery, CPR
should still be performed.
 Idiopathic pulmonary fibrosis: azathioprine, prednisone, n-acetylcysteine combination: higher mortality and side
effects; IPF is a disease with ongoing fibrosis, limited inflammation. Steroids do little to affect progression of fibrosis.
TNF-a etanercept failed to show benefit. Novel therapies for IPD include nintedanib (TKI) which moderates
production of fibrogenic growth factors, and pirfenidone, which modulates production of fibrosis. Both slow decline
in PFT but do not affect QOL, and are not curative.
 Mass in anterior mediastinum: thyroid, thymoma, teratoma (germ cell tumor), terrible lymphoma. Thymoma is most
common in adults, but usually 40-50yo and associated with paraneoplastic syndrome such as MG. lymphoma is
second most common, and affects younger patients (20-30yo). Hodgkin lymphoma is MCC of mediastinal lymphoma,
followed by lymphoblastic lymphoma  primary mediastinal diffuse large B cell lymphoma.
 Posterior mediastinum mass: usually neurogenic tumor. In kids, arise from sympathetic ganglia (i.e. neuroblastoma),
but in adults, arise from nerve sheaths (i.e. schwannoma).
 Bronchogenic cysts: congenital anomaly, develops in middle mediastinal compartment, most 20 yo pts. Usually found
incidentally, rounded lesions, but may be symptomatic if large 2/2 compression. Other middle mediastinal masses
include: pericardial cysts, esophageal duplication cysts.
 For ARDS, manage fluid volume status with conservative strategy, targeting CVP < 4. Large multicenter trials have
shown that conservative fluid strategy leads to shorter mechanical ventilation times, and improvement of lung
function and shorter ICU stay. No effect on mortality. Liberal fluid strategy targets CVP = 12.
 Diagnose exercise-induced bronchospasm and possible asthma with bronchial (methacholine) challenge testing, which
will confirm the degree to which patient’s symptoms are caused by hyperreactivity of lungs. RF: hx of allergies,
cough without exercise, symptoms occurring several times/week. In patients with asthma, spirometry testing between
acute episodes is often normal. Still need to confirm diagnosis in patients with frequent symptomatology.
 In COPD patients with hypercapnia, extubation followed by NIPPV may decrease ICU LOS and improve survival.
 ICU acquired weakness includes: critical illness polyneuropathy with axonal nerve degeneration and critical illness
myopathy with muscle myosin loss, which both result in profound weakness, that may impair vent weaning. Difficult
to discriminate the two types clinically, unless you do biopsy or EMG studies. Both associated with long term
functional disability, prolonged ventilation and higher inpatient mortality. RF: female, hyperglycemia, sepsis, MODS,
SIRS, immobility, long duration of mechanical ventilation. Strategies to limit/prevent include: sedation vacation, early
mobilization, and glucose control.
 GBS: MCC acute diffuse neuromuscular paralysis. Rapid onset of symmetric weakness in upper/lower limbs over
days-weeks in setting of recent infection/trauma/surgery. Progresses over 2 weeks, and >90% patients are at their
worst by 4 weeks. Patients describe paresthesia/neuropathic pain in hands/feet. Weakness throughout, and decreased
or absent DTR.
 LAM: lymphangioleiomyomatosis: uncommon disorder, associated with TSC, seen in women during childbearing
years; histopath shows infiltration of atypical smooth muscle cells into pulmonary interstitium caused by activation of
mTOR (mammalian target of rapamycin) signaling pathway, which results in development of numerous thin-walled
cysts scattered through pulmonary parenchyma and increases risk of spontaneous PTX. CT shows diffuse cystic
changes and serum shows elevated VEGF-D levels. Hormone therapy is not effective. Sirolimus (mTOR inhibitor)
has shown promise in stabilizing pulmonary function.
 Organizing PNA: noninfectious diffuse parenchymal lung disease; usually is associated with collagen vascular disease
or certain meds, but may occur in absence of other condition/exposure (cryptogenic organizing pneumonia). Occurs
over 4-6 weeks; symptoms rarely persist >6 months. Presentation may mimic CAP. CXR shows patchy airspace
disease with consolidation and GGO, but without cystic changes.
 PLCH: pulmonary Langerhans cell histiocytosis: increased Langerhans and inflammatory cells in lung interstitium,
presents with nonspecific respiratory symptoms and hx of spontaneous PTX. Usually 20-40yo, strongly associated
with smoking. Duration of symptoms <1 year before diagnosis is made. Numerous thick walled pulmonary cysts
accompanied by interstitial thickening and nodularity on CT.
 Respiratory bronchiolitis associated ILD: inflammation of bronchioles, primarily in smokers. CT shows centrilobular
nodules with air-trapping and scattered ground-glass attenuation.
 Carbon monoxide poisoning: carboxyhemoglobin serum measurement in ABG. Symptoms: headache, confusion,
nausea/vomiting, LOC, MI, AGMA 2/2 LA, low BP, normal pupils. Pulse oximetry compares light absorption of
oxygenated and deoxygenated Hgb to calculate percentage of Hgb saturated with O2, but isn’t able to detect presence
of methemoglobin or carboxyhemoglobin.
 Sympathomimetic intoxication (cocaine, amphetamines, ephedrine, caffeine): HTN, tachycardia, fever, diaphoresis,
mydriasis, rhabdo. Tx with benzos (1st line therapy) prn.
 Physostigmine: antidote for anticholinergic toxicity (presents as agitation, mydriasis, fever, tachycardia, anhidrosis,
dry throat, dry mouth, urinary retention, constipation, AMS, respiratory depression).
 Anticholinergic toxicity mnemonic: blind as a bat (mydriasis), red as a beet (flushing, vasodilation), hot as a hare
(hyperthermia), dry as a bone (dry skin), mad as a hatter (hallucination, agitation), bloated as a toad (ileus, urinary
retention), the heart runs alone (tachycardia).
 Cocaine induced HTN: BB may paradoxically worsen BP given unopposed alpha (loss of beta mediated vascular
smooth muscle relaxation). Avoid propranolol in sympathomimetic intoxication in general.
 Workup for newly diagnosed lung CA. CT HCAP + bone scan or PET/CT plus MRI brain w/contrast. If mediastinum
and distant sites are negative, surgical resection is best for localized disease. Next step is to determine extent of the
resection and whether postoperative residual lung function will be adequate with PFT to evaluate TLC, FEV, DLCO.
5 year survival rate in patients with resectable disease is 30-75%. If not surgical candidate, then do radiation therapy.
For advanced NSCLC disease with mediastinal, hilar, or contralateral LN involvement, chemoradiation therapy,
where as metastatic distant sites are treated with chemo alone.
 Sarcoidosis: multi-organ systemic inflammatory disease, characterized by tissue infiltration by mononuclear
phagocytes, lymphocytes, and non-caseating granulomas. Etiology unclear, may be 2/2 interaction between HLA2
molecules and T cell receptors. >90% of patients have pulmonary involvement, PFT shows restriction but obstruction
may also be seen. Diagnosis of exclusion based on histology (transbronchial biopsy with endobronchial biopsies).
Usually often spontaneously resolves. Steroids is treatment of choice, but would wait until diagnosis is confirmed.
 CBC recommends IGRA in all settings where TST is recommended. IGRA is as sensitive, but more specific than TST
for diagnosing TB. Testing for TB with both IGRA and TST is not recommended. ACE levels are elevated in 75% of
chronic sarcoidosis, but it lacks specificity, and is limited use for diagnostic function.
 Lofgren syndrome: hilar lymphadenopathy, acute oligoarthritis, erythema nodosum.
 Heerfordt syndrome: uveitis, parotidis, fever.
 Contraindications to BIPAP: AMS/encephalopathy, not able to manage oral secretions, bulbar dysfunction, ineffective
gag or cough reflex, hemodynamic instability, severe acidosis, arrhythmias, active UGIB, airway obstruction, recent
facial trauma/surgery, esophageal/transphenoidal surgery.
 An enlarging pulmonary nodule in high risk patient with high pretest probability for lung cancer warrants a more
aggressive evaluation with tissue biopsy for diagnosis or excision. Surgical resection is best if she is a candidate.
 A negative transthoracic needle aspiration would not be reassuring in a high risk patient, given that the yield is low
(few number of cells), and would require a second diagnostic procedure. Surgical resection is curative and diagnostic.
 Indicators for activating RRT: change in vitals (HR<30, HR>139, SBP<70, SBP>200, RR<9, RR>35, T<34, T>49,
O2<85, AMS, new neurologic changes. s/s of deterioration are present several hours before arrest (median time of 6
hours). RRT helps to prevent chance of cardiopulmonary arrest in decompensated high risk patients.
 Multiple sleep latency testing (MSLT): lab based sleep test, objectively measures sleepiness, used to diagnose
narcolepsy or idiopathic hypersomnia. Shouldn’t be performed in sleep-deprived patients.
 Zolpidem/Ambien: hypnotic medication; sleep aid. SE: sleep driving, sleep eating.
 Acute hypersensitivity pneumonitis (HP): immunologic response to repetitive inhalation of antigens. Presents within
48 hours after high level exposure, associated with fever, flu like symptoms, cough, SOB. CT shows BL hazy
opacities, GGO, centrilobular micronodules that are upper/middle lobe predominant. Symptoms wane after 24-48
hours upon removal of exposure. Recurrence of symptoms with repeated exposure to the antigen.
 Acute interstitial pneumonia (AIP): uncommon form of DLI (diffuse lung injury) that develops days to weeks in
response to unknown insult. Similar to ARDS w/ severe progressive hypoxia and diffuse BL air space opacities.
 Pulmonary rehab: recommended for all symptomatic COPD with FEV1 <50%, s/p acute COPD XC hospitalization.
 Long term O2 therapy: PO2 <55, or SaO2 <88 on room air (confirmed twice over 3 week period). Other indications
include pulmonary HTN, peripheral edema 2/2 RHF, polycythemia, with PO2 <60 or SaO2 <88.
 Restricting blood tx for Hgb <7, reduces risk of cardiac events, rebleeding, bacterial infections, and total mortality.
 Erythropoiesis stimulating agents (ESA) should be considered for CKD patients with symptomatic anemia if Hgb<10.
Before attributing anemia 2/2 CKD, rule out IDA (common in CKD patients). ESA are associated with higher risk of
thrombotic and cardiovascular events and increased BP; it is generally not appropriate for critically ill patient.
 OSA: low lying soft palate, thick neck, male, obese, HTN, obstruction causing postoperative complications,
manifesting as repeated episodes of hypoxia and frank respiratory failure requiring repeated intubations.
 Overnight pulse oximetry has not been validated as method for evaluating OSA.
 Screening tests for OSA: STOP-Bang questionnaire, sleep apnea clinical score, Berlin questionnaire.
 Post-intensive care syndrome: new or worsening function in physical, cognitive, or mental domains that persist after
hospital discharge following a critical illness. Examples include: cognitive deficits resembling TBI or MCI,
psychiatric symptoms such as depression, anxiety, PTSD; physical deficits such as weakness and fatigue. Possible
mechanisms include: hypoxia, hypotension, inflammation, catabolic state, hypoglycemia, nutritional imbalance,
immobility, sedation agents, etc. symptoms may also be applied to close family members.
 Acute embolic mesenteric ischemia: abrupt onset of periumbilical abdominal pain, emesis, and forceful bowel
evacuation, and hypotension, with unremarkable abdominal exam, in a patient with AF. Caused by inadequate blood
flow to small bowel: SMA embolism, SMA thrombosis, nonocclusive mesenteric ischemia 2/2 splanchnic
vasoconstriction, mesenteric venous thrombosis, and focal segmental ischemia. SMA embolism from LA/ventricular
mural thrombi is MCC. May also see high WBC, AGMA 2/2 LA, and negative KUB. High clinical suspicion.
Diagnose with angiography, and consult surgery for resection/intervention.
 Colonic ischemia is 75% of all intestinal ischemia. Bleeding occurs within few days of onset; vomiting helps rule out.
 A higher PEE in ARDS improves lung compliance, VQ matching, and oxygenation. A 2013 systematic review found
no survival difference in patients using higher PEEP (15 cm H2O) vs lower PEEP (8 cm H2O). patients with most
severe ARDS had improvement in oxygenation with higher PEEP. Use PEEP that achieves FiO2 <60% that does not
cause hypotension.
 Pneumococcal vaccination with PPSV23 should be given to all patients 19-64yo with COPD, with revaccination at
65yo if >5 years have elapsed since previous pneumococcal vaccine. All patients with/without COPD should get the
13 valent conjugate PCV13 at 65yo. Ideally, PCV13 should be given >1 year after PPSV23. PPSV23 and 13 are also
indicated for patients with asplenia, cochlea implants, persistent CSF leak, and immunocompromising conditions.
These patients should get PCV13 first, followed by PPSV23 8 weeks later. COPD patients who received their first
PPSV23 >65 do not need revaccination. PCV13=Prevnar. PPSV23=Pneumovax.
 For auto-PEEP causing hemodynamic instability, disconnect patient from ventilator for a few seconds to allow
expiration and intrathoracic pressure release (which caused the VR/CO); once patient is reconnected to vent,
slowing the respiration rate, decreasing the tidal volume, and increasing the inspiratory flow rate while tolerating
respiratory acidosis helps to increase the exhaled volume with each respiratory cycle.
 Lung protective strategy in ARDS: maintain low tidal volume (6 mL/kg of IBW), low plateau pressure <30,
conservative fluid strategy to keep lungs dry, prone positioning, early therapeutic paralysis for first 48 hours if
severely hypoxemia to help with vent synchrony. Permissive hypercapnia is acceptable if it means ventilating at lower
tidal volumes and plateau pressures, despite reduction in arterial pH. A pH as low as 7.25 is OK for severe ARDS.
 IPF (idiopathic pulmonary fibrosis): RF (smoking, organic dust exposure, age). Prevalence of IPF increase with each
decade of life >50. Progressive DOE and cough, exercise intolerance. CT findings consistent with UIP (usual
interstitial pneumonia- peripheral/basal predominant septal line thickening and honeycomb changes), which is the
pathologic correlate of IPF.
 Respiratory bronchiolitis associated ILD: strongly associated with smoking. CT shows middle/upper lung
predominant centrilobular micronodules.
 High altitude pulmonary edema: mechanism 2/2 noncardiogenic exaggerated hypoxic vasoconstriction of pulmonary
vasculature, usually when >2500 meters (>8200 feet). Symptoms: cough, dyspnea, exercise intolerance, headache,
fatigue, nausea/vomiting, disturbed sleep. Pink frothy/foamy sputum, or frank hemoptysis; crackles on exam. Tx with
supplemental O2 and rest. Descent from altitude and pulmonary vasodilators (nifedipine, PDE5 inhibitors-sildenafil)
are recommended for first line; nifedipine: relaxation of vascular smooth muscles.
 Acute mountain sickness: acetazolamide, dexamethasone, O2 therapy, ibuprofen.
MKSAP: NEPHROLOGY
 Prehypertension is 120-140/80-90 (JNC7). Consider DASH (dietary approaches to stop HTN) diet prior to Rx.
 Start with US to evaluate painless gross hematuria to rule out obstructive causes such as cancer/tumor or structural
lesions. Avoid CT and MRI for CIN and NSF respectively.
 Balkan endemic nephropathy: increased risk for TCC (transitional cell CA-renal pelvis/ureter/bladder) 2/2 exposure to
Aristolochic acid. BEN: slowly progressive tubulointerstitial disease. Aristolochic acid: nephrotoxic alkaloid plant
aristolocholia clematis; an herbal therapy for weight loss.
 Tubulointerstitial disease: polyuria, decreased concentrating ability, glucosuria without hyperglycemia, tubular
proteinuria.
 Mechanical MV: MAHA w/thrombocytopenia. Free Hgb is partially bound to haptoglobin, and partially filtered by
kidneys, causing hemoglobinuria. Heme reacts with peroxidase in the dipstick, causing FP blood, given lack of RBC
on the microscopy. Similarly: rhabdo causes myoglobinuria (FP blood) with negative RBC, and red urine. Myoglobin
is not bound to haptoglobin, and freely filtered by kidneys.
 Resistant HTN: high BP despite 3 Rx, one of which must be a diuretic. Consider chlorthalidone for resistant HTN
given longer duration of action and higher potency over other thiazides.
 Minoxidil is vasodilator, but causes sodium retention, so is given in combination with a diuretic.
 AL amyloidosis (monoclonal lambda or kappa light chains), a plasma cell dyscrasia. Amyloids: random fibrils that
form organized beta pleated sheets in tissues. May present as nonspecific weight loss/fatigue, or symptoms associated
with infiltration of organs: restrictive cardiomyopathy, peripheral neuropathy, hepatosplenomegaly, cutaneous
purpura, macroglossia. Renal amyloidosis: nephrotic range proteinuria, glomerular lesions, autonomic symptoms
(postural hypotension, diarrhea), deposition in tubular basement membranes, interstitial spaces, and blood vessels.
Biopsy reveals deposition that stain Congo red with apple green birefringence.
 Myeloma nephropathy: tubular cast nephropathy 2/2 filtering of myeloma light chains. Dx with UPEP.
 Type 4 RTA (hyperkalemic distal): caused by aldo deficiency vs resistance. Primary aldo deficiency is 2/2 Addison’s
disease (primary adrenal insufficiency, hypocortisolism), and relative aldo deficiency may be seen in hyporeninemic
hypoaldosteronism from: diabetic nephropathy, chronic interstitial nephritis (2/2 HIV/SLE), acute GN, drugs that
impair renin release (NSAIDs, calcineurin inhibitors), tubulointerstitial disease, or drugs that reduce aldo production
(ACEi, COXi, heparin).
 Type 4 RTA presents with hyperkalemia, normal AG, impaired urine acidification, but with urinary pH <5.5. Treat
hyporeninemic hypoaldosteronism with Florinef, unless HF or HTN present.
 Anion gap acidosis in AKI/CKD is 2/2 impaired excretion of organic acids.
 Type 1 RTA (hypokalemic distal): caused by impaired excretion of H ions in distal tubule. Normal AG. hypoK.
 Type 4 RTA (proximal): caused by defect in regeneration of bicarb in proximal tubule. Causes hypoK, glycosuria (in
setting of normal serum glucose), low molecular weight proteinuria, and renal phosphate wasting.
 Primary Addison’s disease (adrenal insufficiency) is usually 2/2 autoimmune process (developed countries) vs TB
(developing world).
 Secondary adrenal insufficiency: not enough CRH (hypothalamus) or ACTH (pituitary).
 Glomerular hematuria: brown/tea colored urine with dysmorphic RBC (acanthocytes) and RBC casts on urine
sedimentation exam. May also see proteinuria. Caused usually by inflammatory processes (GN), vs more indolent
benign diseases such as thin GBM disease, IgA disease, and other chronic GN.
 IgA nephropathy/Berger Disease/synopharyngitic glomerulonephritis: asymptomatic, normal renal function, recurrent,
or episodic gross hematuria. Related to HSP. Starts few days after URI. IgA deposition within glomerulus. MCC of
GN. Normal complement levels. Common in 20-30 yo. M>F.
 Thin GBM disease: inherited type 4 collagen disease; causes thinning of GBM and hematuria. +family hx. Excellent
prognosis, few progress to CKD.
 Fabrey disease: X linked. Deficiency of alpha-galactosidase A (enzyme in glycosphingolipid pathway), which leads to
progressive deposition of globotriaosylceramide (Gb3) in lysosomes. Presents as CKD in young adulthood. Also
associated with CAD, severe neuropathy, telangiectasia, and angiokeratomas.
 Hereditary nephritis: X linked usually. Alport Syndrome: type 4 collagen defect. Associated with SNHL, lenticonus
(conical deformation of lens), proteinuria, HTT, ESRD.
 TSC: tuberous sclerosis complex: mutations in genes for proteins that have tumor-suppressing effects. Leads to
abnormal cell proliferation in various tissues (skin, brain, lung, liver, kidney). Renal angiomyolipoma.
 Stage 1 HTN: 140-160/90-100.
 Start ACEi/ARB in patients with HTN and CKD, even if not diabetic.
 JNC8: BP goal for HTN with DM: <140/90. BP goal for CKD: <140/90.
 JNC8: BP goal for >60yo: <150/90
 alpha blocker doxazosin usually be used for dual disease: HTN/BPH, but SE includes orthostatic hypotension, and
increased incidence of HF as noted in the ALLHAT (antihypertensive and lipid lowering treatment to prevent heart
attack) trial.
 Careful with use of diuretics in patients with gout/HTN; precipitates gout attack. Consider ARB, uricosuric effect.
 BB no longer first line tx for HTN. For newly diagnosed, consider ACEi/ARB/CCB.
 IRGN: infection related GN: nephritic urine sediment (erythrocytes, RBC casts, proteinuria) in azotemic patient with
active infection (usually non-strep). Immune complex mediated disease; deposition of IC in subepithelial cells,
leading to inflammation and proliferative GN. Low C3, absent cryoglobulins. Tx underlying infection. No steroids.
 Drug induced tubular toxicity (e.g. vanco), occurs 7-10 days after therapy; sediment negative for cells.
 AIN 2/2 Abx: UA shows mild proteinuria, RBC, WBC, WBC casts (eosinophils). Also may have fever, rash, and
peripheral eosinophilia 7-10 days after therapy.
 For young patients with microscopic hematuria, monitor serial P7 and urine proteins; they have higher incidence of
mild glomerular disease (IgA nephropathy, genetic collagen disease) especially with positive family history, whereas
older patients have higher incidences of stones, cancer, and infections.
 Young female with asymptomatic microscopic hematuria: consider familial hematuric syndrome. X linked mutation
of type 4 collagen. Kidney failure may occur later in life. Serial monitoring. No consensus on frequency of labs.
 RF for GU CA: smoking, analine dye, cyclophosphamide.
 When to start BP meds: AM BP >135/85 or PM BP>125/75. Physiologic BP dip from AM to PM.
 Non-dipping BP: independently associated with LVH, CV events, moderately increased microalbuminuria, and more
rapid decline of GFR. Consider starting pharmacotherapy if indicated on ambulatory BP measurements.
 Uric acid kidney stones: step 1: increase free water PO intake (>2L/d). Step 2: urinary alkalinization with potassium
citrate or potassium bicarbonate to increase solubility of uric acid. RF: dehydration and acidic urine, DM, gout,
hyperuricemia, metabolic syndrome, diarrhea. Allopurinol (xanthine oxidase inhibitors) is for patients who are
refractory to fluids and alkalinization, or with urinary uric acid >1000mg/day.
 Oxalate acid kidney stones 2/2 hyperoxaluria: treat with cholestyramine to bind bile salt and oxalate in gut.
 Calcium kidney stones: treat with thiazide diuretics, to decrease hypercalciuria (via increased proximal sodium
reabsorption with passive reabsorption of calcium).
 In CKD4 (GFR<30), thiazide diuretics lose potency; switch to loop diuretic for better diuresis.
 Elderly patients with ESRD and high burden of comorbid disease with poor functional status may live just as long, or
longer with non-dialytic therapy that is focused on alleviating symptoms and maximizing QOL.
 Treat stage 2 HTN (>160/100) with dual therapy: lisinopril + Norvasc (ACE+CCB), or thiazide+ACE/ARB. A single
rx is unlikely to control BP in patients who have BP >20/10 above target pressures.
 The Avoiding Cardiovascular Events in Combination Therapy in Patients Living with Systolic HTN (ACCOMPLISH
Trial) showed reduced cardiovascular events in patients with ACE+CCB compared with ACEi+thiazide.
 Diagnose white coat HTN with ambulatory BP monitoring (ABPM). Increases risk for HTN in future.
 Pheochromocytoma triad: sweating, headache, and palpitations/tachy.
 Aminoglycoside induced AKI (gentamycin): presents as non-oliguric ATN with granular casts and FeNa >1%. Occurs
5-10 days after therapy. Hypokalemia/hypomagnesemia also seen 2/2 renal electrolyte wasting.
 Cephalosporins usually cause AKI 2/2 AIN. UA: WBC, WBC casts, eosinophilia, RBC. Also p/w fever, rash, and
peripheral eosinophilia. Occurs 3-5 days after second exposure, and weeks to months with first exposure.
 Renal transplant patients require lifelong immunosuppression. Monitor for metabolic complications: DM/HLD.
 NODAT: new onset diabetes after transplantation. RF: steroids, tacrolimus/prograf, mTOR-mammalian target of
rapamycin inhibitors (sirolimus, everolimus).
 Cyclosporin and mTOR inhibitors also lead to dyslipidemia.
 ESRD leads to secondary hyperparathyroidism, which causes hypophosphatemia.
 If kidney stone is >10mm, then mechanically remove.
 If kidney stone is <10mm, then give hydration, analgesia, tamsulosin/Flomax or calcium channel blocker (nifedipine)
for expulsion therapy. +/- steroids. May use mechanical removal if <10mm but refractory to medical therapy.
 Mechanical removal of stones: renal pelvis (shockwave lithotripsy), ureter (shockwave lithotripsy vs ureteroscopic
lithotripsy). Impacted stones: percutaneous antegrade ureteroscopy or retroperitoneal laparoscopy). Also consider
mechanical removal for complicated stones (urosepsis, AKI, anuria, refractory pain).
 IgG4-related interstitial nephritis: autoimmune disease. UA consistent with tubulointerstitial process (mild
proteinuria, inflammatory cells-WBC, RBC, WBC casts). Bx shows IgG4 plasma cells in renal tissue. Tx: steroids.
 IgG4 related disease: autoimmune pancreatitis, interstitial nephritis. Caused by systemic infiltration of organ systems
by lymphoplasmacytic IgG4 positive plasma cells, which results in end organ fibrosis; high IgG4 serum levels. May
present with kidney and liver failure, or with mass-like lesions on kidney and liver. Organs may also appear enlarged
2/2 inflammatory process from cellular infiltration. Tx with steroids.
 Inflammatory process: WBC and WBC casts. Vasculitis process: RBC and RBC casts (GN).
 Barter syndrome: AR disorder of sodium/chloride transporters. Mimics loop diuretics.
 Gitelman syndrome: AR disorder of sodium/chloride transporters. Mimics thiazide diuretics.
 Cushing syndrome: prox muscle weakness, hypokalemia, HTN, DM. Dx with 24 hour free cortisol in urine.
 ACEi decrease angiotensin 2, which decreases arterial BP as well as causes efferent arteriolar dilatation, causing
lower GFR, which increases creatinine up to 25-30%.
 Vaccination for CKD: give PCV-13 (conjugate) >1 year after giving PPSV23 (polysaccharide). Second dose of
PPSV23 should be given 5 years after first PPSV23 dose.
 Who else should get pneumonia vaccine: asplenia, congenital/acquired immunodeficiency, HIV, nephrotic syndrome,
cirrhosis, cancer, MM, transplantation, immunosuppressed patients.
 For CKD <65yo who haven’t had pneumo vaccine, PCV13 should be given first, followed by PPSV23 >8 weeks later.
 Non-AG acidosis: either renal vs GI losses. Urine AG (Na+K-Cl). NeGUTive. PISSative. Normally, acidemia causes
body to excrete acid in urine, causing increase in urine ammonium. Ammonium isn’t directly measured, but attracts
chloride into urine to maintain electro neutrality. Amount of urinary chloride reflects amount of ammonium excreted.
 RTA: non-gap acidosis with positive urinary anion gap.
 Renal tx patients are at higher risk of malignancies (SCC of skin and post-transplant lymphoproliferative disease).
SCC is more common than BCC. SCC accounts for 90% of skin CA in transplant patients. Also more likely mets.
 Oncogenic osteomalacia: usually 2/2 benign mesenchymal tumor of vascular/skeletal origin. Tumor overexpresses
FGF23, which leads to decreased resorption of phosphate in renal tubules, causing hypophosphatemia and
hyperphosphaturia.
 Calcitriol: active Vitamin D: 1,25-dihydroxycholecalciferol. 1,25-dihydroxyvitamin D. increases calcium resorption
from gut and kidneys, and increases release from bone, to increase serum calcium. Acts on osteoblasts to upregulate
RANKL which activates osteoclasts to break down bone. Will see low 1,25-DHC, but normal 25-hydroxy vit D.
 Chronic hypophosphatemia causes abnormal mineralization of bone, increased alk phos, osteomalacia, and fractures.
 Nutritional Vit D deficiency causes: low calcium, low 25 hydroxy vitamin D concentration.
 Primary hyperparathyroidism: elevated PTH causes decreased phosphate, increased calcium, and increased 1-25 DHC
 PTH upregulates 1-a-hydroxylase, which metabolizes 25-hydroxyVD into 1-25 DHVD, which is the active from that
is responsible for increasng gut uptake of calcium. PTH also reduces resorption of phosphate in kidneys. PTH causes
bone resorption to break down calcium via binding to osteoblasts, increasing their expression of RANKL, which binds
to RANK receptors on osteoclasts.
 CRRT: continuous renal replacement therapy (24h/day): modality for slow dialysis for hyperK, hyperPhos, AGMA,
etc, for patients who have hemodynamic instability. Venovenous access. Either with diffuse based solute removal
(dialysis) or convection based solute and water removal (filtration).
 Pulmonary renal syndrome ddx: small vessel vasculitis, microscopic polyangiitis (ANCA), anti-GBM disease
(Goodpasture), other autoimmune (SLE, cryoglobulinemic vasculitis, IgA vasculitis), microscopic polyangiitis
 Goodpasture disease: antibodies against noncollagenous domain of type 4 collagent that binds to the GBM. Leads to
inflammatory response and damage, with proliferative and crescentic GN. Same process occurs in the BM of pulm
capillaries, leading to alveolar hemorrhage. Serum shows normal complement levels, with elevated anti-GBM Ab.
Renal biopsy shows necrotizing crescentic proliferative GN, with linear deposition of Ig along the GBM. Tx
cyclophosphamide and steroids, with daily plasmapheresis to remove the anti-GBM Ab.
 Nephrotic syndrome: MCD, membranous GN, FSGS, RPGN. Low albumin, loss of protein in urine. Complications:
infections, VTE, anasarca, etc.
 Microscopic polyangiitis is MCC of pulmonary renal syndrome, which shows +ANCA with vascular lesions
(purpura). Kidney biopsy shows little/no immune deposition (“pauci-immune GN”).
 Creatinine is filtered through kidney from serum; proximal tubule also secretes creatinine.
 In patients with high clinical suspicion of nephrolithiasis and negative US, get CTAP/stone protocol. US is less
sensitive for stones in distal ureter.
 Bactrim: TMP/SMX: TMP interferes with Cr secretion without affecting GFR. May cause increase in serum Cr up to
0.5; this rise does not reflect drop in renal function. Resolves after completing abx therapy. No need to d/c. TMP may
also inhibit ENac (epithelial Na channels) in collecting tubule, acting as potassium sparing diuretic, which may cause
hyperkalemia. Beware in CKD with hyperkalemia, or in patients already on ACEi/ARB.
 AKI 2/2 pre-renal azotemia is either from decreased intravascular volume, or diminished effective circulation.
 FeNa <1% and FeUrea <35% is suggestive of pre-renal disease. Beware: patients on lasix and falsely elevated FeNa.
 Lupus nephritis: class 3-5 on biopsy: start on prednisone and immunosuppression. LN occurs in 80% of SLE patients.
 Lupus nephritis: class 1-2, asymptomatic, watch and wait. Class 3-4, nephritic symptoms, reasonable to start
immunosuppressive therapy (prednisone, cellcept/mycophenolate mofatil), class 5, nephrotic syndrome, prominent
protineuria. Class 6, end stage, with fibrosis and renal failure, no role for biopsy or therapy.
 For lupus nephritis flare: pulse steroids for 3 days (IV solumedrol 1mg/kg/day), then PO prednisone. For more severe
disease, may consider cyotoxan (cyclophosphamide), especially if CNS or cardiac involvement.
 Class 4 lupus nephritis: diffuse glomerular involvement. Most common and severe form of nephritis associated with
lupus. Low complement (C3, C4) levels (2/2 classical pathway activation), and high anti-ds DNA titers. Biopsy:
diffuse proliferative GN with immunofluorescence showing granular deposition in subendothelial, mesangial, and
subepithelial areas (IgG, IgM, IgA, C3, C1q).
 PSGN: preceding URI (rapid strep Ag test, anti-SLO Ab), nephritic syndrome (HTN, proteinuria, RBC casts, ATN,
hypoalbuminemia) after 1-2 weeks, low C3, normal C4 (2/2 alternative pathway of complement activation), usually
with spontaneous resolution of GN s/p conservative tx (BP monitoring, Abx, diuretics).
 Oxalate crystal nephropathy 2/2 orlistat via enteric hyperoxaluria. Exacerbated in CKD or dehydrated patients.
 Orlistat: blocks fat uptake from gut, which results in production of calcium soaps from unabsorbed fat in small bowel.
Calcium soaps reduce availability of free enteric calcium, preventing binding of oxalate in gut, allowing for increased
intestinal uptake of oxalate and subsequent renal oxalate excretion. Excess Hyperoxaluria leads to development of
oxalate crystals within renal tubules and interstitium, causing obstruction and AKI.
 Aristolochic acid: extensive interstitial fibrosis with tubular loss, seen on renal biopsy. Weight loss supplements. Also
at higher risk of urothelial malignancies.
 Ephedrine: associated with nephrolithiasis (ephedrine, norephedrine, pseudoephedrine stones). May also have rhabdo.
 Membranous glomerulonephritis: nephrotic syndrome, higher risk than other nephrotic entities to cause VTE, in
particular renal vein thrombosis. Ppx anticoagulation is frequently given to MG patients with Albumin <2.8. MCC
idiopathic nephrotic syndromes in white adults. MG also associated with infections, SLE, meds, and malignancies.
 Thrombotic microangiopathy (TMA): MAHA, thrombocytopenia, AKI. Caused by chemo meds: mitomycin C,
gemcitabine, TKI, mTOR inhibitors (sirolimus, everolimus), VEGF-antivascular endothelial growth facor inhibitors
(bevacizumab, sunitinib). VEGF inhibitors cause hypertension and renal dysfunction 2/2 induced microangiopathy by
inhibiting the growth of blood vessels.
 All patients with CKD G4-5 need renal involvement for starting dialysis. GFR<20, discuss transplantation options.
HD and PD have similar outcomes, mortality rates.
 Max doses of ACEi/ARB should be given in slowing progression of diabetic nephropathy (reducing proteinuria), as
the patient’s BP can tolerate. Offers long term renoprotection. Dual ACEi+ARB causes AKI and hyperK. Don’t do it.
 Effects of angiotensin 2:
 Angiotensinogen (hepatocytes)converted by renin (kidneys) to angiotensin 1converted to angiotensin 2 by
angiotensinogen (pulmonary and renal endothelial cells).
 Angiotensin 2 results in: increased sympathetic activity, arterial vasoconstriction, efferent>afferent renal constriction,
causing GFR, ADH secretion in posterior pituitary (which increases free water absorption in collecting ducts), aldo
secretion from the adrenal cortex.
 Suspect abdominal compartment syndrome in patients with oliguria/ARF who have recently had abdominal surgery,
tense ascites, massive fluid transfusion, or liver/pancreatic disease. Intra-abdominal pressures >20. Tx: surgery and
medical therapy: diuretics, hemodialysis, and ascites management.
 AKI 2/2 statin induced rhabdo: UA with blood, but negative for RBC. Myoglobin causes direct nephrotoxicity, renal
tubular obstruction, intrarenal vasoconstriction, and AKI. RF for statin induced rhabdo: old, female, CKD, DM,
hypothyroidism, high statin dose, other rx metabolized through cytochrome p450 system.
 ACKD (acquired cystic kidney disease): Innumerable small BL cysts in kidneys. Incidence of ACKD rises
proportional to number of years spent on HD (ESRD). ACKD is 30x risk of developing RCC.
 ADPKD: cysts usually originating in renal collecting duct. +FHx for PKD/CKD. Hemorrhagic kidney cysts is usually
indistinguishable from solid renal mass on US.
 Transitional urothelial CA: mucosal surface of urethra, bladder, ureter, renal pelvis/calyces.
 In patients with diabetic CKD and microalbuminuria, ACEiARBs can prevent progression to overt nephropathy.
 Dipstick is not a sensitive test: results are semiquantitative, and estimates of proteinuria is affected by urine conc.
 Beware of chronic tubulointerstitial disease 2/2 interstitial nephritis from PPI use.
 In acute CHF XC, elevated renal venous pressures causes increased tubular fluid pressures, leading to venous
congestion and CRS 1 (cardiorenal syndrome).
 CRS 1: acute CHF XC leading to AKI
 CRS 2: chronic CHF leading to CKD
 CRS 3: AKI leading to acute CHF XC
 CRS 4: CKD leading to chronic cardiac dysfunction (CHF, CAD, arrhythmias)
 CRS 5: systemic conditions leading to simultaneous cardiac/renal dysfunction (sepsis, shock).
 In CRS 1: treatment is directed towards improving cardiac fx and fluid balance; it’s common to see moderate
worsening of renal function associated with diuretics until fluid balance is achieved. Despite AKI, you should
continue diuretics until CVP/edema is resolved, even if there is asymptomatic mild/moderate Cr bump.
 Excessive concern about AKI in CHF XC patients leads to underutilization of diuretics and persistent volume
overload state, which reduces efficacy of ACEi and increases risk of coreg induced decompensated HF.
 HTN <20w gestation is consistent with chronic HTN. >20w gestation is consistent with gestational HTN. Per ACOG,
tx if BP>160/100. Gestational HTN resolves within 12 weeks after delivery.
 Pre-eclampsia: HTN >20weeks, with end organ damage (proteinuria, renal dysfunction, thrombocytopenia, LFT,
pulmonary edema, cerebral or visual symptoms). RF: DM, FHx, Obesity, CKD, twin gestation, advanced maternal
age, first pregnancy.
 Hyperosmolar hyponatremia 2/2 glucose (1.6 increase in every 100 above 100).
 Pseudohyponatremia: hyponatremia 2/2 hyperlipidemia and hyperparaproteinemia. Normal plasma osm (lipids and
proteins are not measured in osm).
 Hypo-osmolar hyponatremia: SIADH, hypothyroidism, adrenal insufficiency. High adh, low serum osm.
 Reset osmostat: downward setting of level at which sensors of plasma osm triggers release of ADH. Associated with
patients with quadriplegia, TB, advanced age, pregnancy, psych d/o, and chronic malnutrition. Causes chronic, stable,
but mild hypo-osmolar hyponatremia.
 Hydroxyethyl starch (HES) contraindicated in septic shock, requiring fluid resuscitation. Causes AKI.
 High risk patients for CIN (>75, DM, CKD, nephrotoxic meds) can be reduced/prevented by giving IV isotonic saline
(1-1.5cc/kg/hour) 3-12 hours prior to contrast, and continued 6-24 hours afterwards.
 Drug induced AIN can occur months after exposure (usually most often with NSAIDs and PPIs).
 Struvite renal stones: composed of magnesium ammonium phosphate. Occurs when ammonium production is
increased, which urine pH and decreases solubility of phosphate. Common complication of chronic upper GU UTI
with urease producing organism (proteus, Klebsiella). Struvite stones can grow rapidly and large, causing staghorn
calculus. <10% of all kidney stones, but occurs more commonly in women with chronic/recurrent UTI. Dietary
phosphate reduction and urine acidification won’t help when stone is large/formed. Remove mechanically.
 HIV infection is associated with collapsing form of FSGS; in early stages ARV and ARB may halt disease
progression.
 Primary FSGS is cause of idiopathic nephrotic syndrome in 25% of cases.
 Secondary FSGS may be hyperfiltration injury to glomerulus such as in: chronic HTN, DM, progressive CKD,
obesity, CSD, reflux nephropathy, s/p nephrectomy. May also be 2/2 drug toxicity (pamidronate, interferon, and
infections-HIV).
 Hep B: membranous glomerulopathy.
 Hep C: cryoglobulinemic glomerulonephritis.
 SPEP/UPEP: monoclonal gammopathies.
 Treponemal antibody: syphilis: membranous glomerulopathy.
 Acute GN (glomerulonephritis): HTN, dysmorphic RBC, RBC casts, proteinuria, inflammation (WBC). If findings of
acute GN with constitutional symptoms (fever, arthralgia, nonblanching rash-palpable purpura, weight loss), think
about underlying vasculitis. Diagnose with renal biopsy.
 Hypovolemia (GI/GU losses of water) leads to increased ADH release by body, causing hyponatremia, especially if
compensated with copious free water intake. IV volume repletion in the hospital suppresses ADH with subsequent
free water diuresis, leading to rapid correction of hyponatremia, which places patient at risk of ODS.
 ODS risk is higher in women, which may have delayed onset a few days after overcorrection. ODS may result in
severe neuro symptoms such as paraplegia, dysarthria, dysphagia, diplopia, locked in syndrome.
 Overcorrection of chronic asymptomatic hyponatremia is >8 in first 24 hours, or >16 in first 48 hours.
 For patients with ADPKD, screening for intracranial cerebral aneurysm using MR angiography is only recommended
for those with +FHx of aneurysm/SAH, personal hx of previous rupture, or those with high-risk occupations in which
a rupture would affect the lives of others (school bus drivers).
 Intracranial cerebral aneurysms (ICAs) can be detected in 10-12% of patients with ADPKD. A ruptured ICA resulting
in SAH is most serious extra-renal complication of ADPKD.
 Demeclocycline results in renal resistance to ADH. May be used to treat SIADH. Careful in CKD or liver disease,
may cause AKI. Reserved for patients who have refractory SIADH.
 Tolvaptan (vasopressin receptor antagonist) results in excretion of free water, used to treat SIADH. Last resort for
SIADH if refractory. Severe liver injury may occur if used for >30 days. FDA limits use to 1 month.
 Patients with primary polydipsia may present with transient ADH release during psychotic episodes, and increased
renal response to ADH (ADH sensitivity). Acutely psychotic patients may have higher urine osm and downward
resetting of osmostat that regulates ADH release. Urine of psychotic patients with primary polydipsia may not be as
dilute as expected.
 Type 1 distal RTA. Hypokalemia. Impaired hydrogen excretion 2/2 dysfunctional proton pumps in tubular cells.
NAGMA with compensatory Hyperchloremia, inability to acidify urine (always pH >6). The same defect in tubular
cells causes potassium wasting, and increased proximal resorption of citrate (as compensatory response to metabolic
acidosis) leads to hypocitraturia and increased risk of calcium phosphate kidney stones and nephrocalcinosis. Type 1
RTA is associated with: genetic, autoimmune d/o, nephrocalcinosis/hypercalciuria, dysproteinemia, drugs, toxins, and
tubulointerstitial disease. Sjogren (AI) causes interstitial nephritis and type 1 RTA.
 Type 2 proximal RTA. Hypokalemia. Defect in regenerating bicarb in proximal tubular. NAGMA, glycosuria
(without hyperglycemia), low molecular weight proteinuria, and renal phosphate wasting. Distal urine acidification
mechanism is intact, urine pH usually <5.5.
 Type 4 RTA: hyperkalemic distal. Urine pH <5.5, hyperkalemia 2/2 hypoaldosteronism.
 Tylenol/acetaminophen: associated with pyroglutamic acidosis (5-oxoprolinuria), which causes HAGMA.
 There is no difference in clinical outcomes (mortality, QOL) between HD and PD. Driven by patient preference.
 There is evidence that risk of graft loss and overall mortality increases in patients who have been treated with dialysis
prior to transplantation; this risk increases in proportion to the length of dialysis prior to transplant.
 Low protein diet may delay onset of symptomatic uremia in late stage CKD (G4-G5), but does not slow progression
of disease.
 Definition of nephrotic range proteinuria: >3.5 g/24h of urine protein, or >3500 mg/g of urine protein/creatinine ratio.
 Rhabdo: hyperkalemia, hyperphosphatemia, hypocalcemia, UA +blood, negative for RBC on microscopy.
 Alcoholic ketoacidosis in chronic alcohol abuse, with liver disease, develops following acute intoxication. During
acute intoxication, ethanol already extensively metabolized (may present with low/normal serum ethanol levels).
 Ethanol is oxidized to acetaldehyde, then to acetic acid, during which process the electron carrier coenzyme NAD+
(nicotinamide adenine dinucleotide) is reduced to NADH in increasing amounts. Simultaneously, increased
catecholamines causes lipolysis with generation of free fatty acids and ketone bodies (acetoacetate). High
NADH/NAD+ ratio leads to increased reduction of acetoacetate to b-hydroxybutyrate.
 IgA vasculitis (HSP-Henoch Schonlein purpura): fatigue, joint pain (no effusion), abdominal pain, petechia, purpura,
GN following URI (within 1-2 days). Tetrad of: palpable purpura, arthralgia, abdominal pain, and GN. No serologic
tests for diagnosis, but normal complement levels, and +biopsy.
 Infection related GN 2/2 IE activates alternative complement pathway: low C3, normal C4 levels.
 Cryoglobulinemic and lupus vasculitis activates classical complement pathway: low C3 and low C4.
 DDx for vasculitis with GN: infection related (IE), cryoglobulinemia, SLE, ANCA associated, IgA.
 Hypokalemia from vomiting is associated with contraction met alkalosis, with increased urine potassium excretion
and decreased urine chloride excretion. Vomiting results in loss of HCL and fluid from gastric secretions, which if
persistent, leads to volume contraction. Hypovolemia leads to RASS activation and increased Na-H exchange and
increased bicarb reabsorption in proximal tubule, which is exacerbated by decreased chloride available for
reabsorption with sodium. Aldo stimulates Na-K exchange in distal tubule. Urine studies show low urine sodium due
to conservation of sodium. Low urine chloride reflects GI loss and prolonged volume contraction which leads to avid
reabsorption of Cl with Na. Urinary K is elevated 2/2 aldosterone
 Bartter syndrome: mimics effects of loop diuretics. Increased urine sodium, potassium, and chloride, all >40.
 Shiga toxin associated HUS. Diarrhea associated syndrome with MAHA, thrombocytopenia, and ARF. Associated
with shiga toxin from E coli O157:H7 strain, and shigella dysenteriae. The toxin binds to endothelial cells, which
triggers thrombosis, causing MAHA. Toxin also directly binds to renal mesangial cells, podocytes, tubular cells.
Treatment is supportive, usually requiring HD. Bx shows capillary thrombosis, with glomerular/tubular damage.
 ADAMTS13: metalloprotease enzyme that cleaves vWF. Low levels of ADAMTS13 support dx of TTP (thrombotic
thrombocytopenic purpura).
 UPEP used to evaluate Ig light chains (bence jones proteins). Plasma cell dyscrasia may cause anemia and AKI 2/2
precipitation of light chains in renal tubule (myeloma kidney).
 Crohn disease, s/p bowel resection, results in enteric Hyperoxaluria, which predisposes to calcium oxalate
nephrolithiasis (Hyperoxaluria). Tx with bile salt binders (cholestyramine) to decrease enteric oxalic acid absorption.
 Hyperoxaluria may be 2/2 excess intake (chocolate, spinach, rhubarb, green/black tea), or 2/2 reduction in dietary
calcium intake, which decreases binding of calcium to dietary oxalate in gut, increasing free oxalate absorption.
 Enteric Hyperoxaluria results from malabsorption, when excess free fatty acids in GI tract binds calcium, increasing
free oxalate absorption in colon. Seen in small bowel resection/disease. Must maintain adequate UOP and adequate
calcium dietary intake. May benefit from cholestyramine.
 Urine citrate inhibits stone formation by binding calcium in tubular lumen, preventing it from precipitating with
oxalate. Citrate excretion in urine can be enhanced by alkalinizing serum with potassium citrate, which decreases
uptake of filtered citrate from tubular lumen.
 Pyridoxine is indicated in some patients with primary Hyperoxaluria, to improve glyoxylate metabolism and reduce
overproduction of oxalate.
 Chronic respiratory alkalosis: serum bicarb drops 4-5 for every 10 drop in CO2.
 Tenofovir induced tubulointerstitial disease presenting as Fanconi syndrome. Slow progressive chronic indolent
course, without clear inciting event, subnephrotic proteinuria, bland urine sediment, and atrophic kidneys on renal US.
 Fanconi syndrome: abnormal tubular handling of glucose, amino acids, uric acid, phosphate, bicarb. RTA also
common. Clinically presents with glucosuria in context of normoglycemia, trace proteinuria, and hypophosphatemia.
 Chronic Hep B may cause membranoproliferative GN (IC deposition in glomeruli), or membranous GN (mesangial
and subendothelial IC deposition).
 Refeeding syndrome: hypophosphatemia. D5 infusion exacerbates hypophosphatemia by stimulating insulin release,
which promotes phosphate uptake in cells. Occurs in chronic alcoholics, malnutrition, and critical illness states.
 Severe symptoms of hypophosphatemia (<1): weakness, myalgia, rhabdo, arrhythmia, heart failure, respiratory
failure, seizure, coma, hemolysis.
 How do alcoholics get hypophosphatemic? Via decreased dietary intake of phosphate and vitamin D, chronic
diarrhea, and direct toxicity to the renal proximal tubular cells (reabsorption of phos).
 All NSAIDs contribute to HTN via inhibition of COX2 (cyclooxygenase) in kidneys, promoting sodium retention and
increased intravascular volume. Other SE of NSAIDs include hyperkalemia, and reduced urine potassium excretion
via reduced RASS (aldosterone promotes renal potassium excretion).
 Lactic acid exists in 2 isomers: D & L. L is measured routinely as marker of hypoperfusion. D-LA is from short gut
syndrome s/p bypass or bowel resection; it manifests as intermittent confusion, slurred speech, ataxia, and AGMA
with normal plasma lactate (L LA).
 D lactate: excess carbs that reach colon are metabolized to D lactate.
 Pyroglutamic acidosis 2/2 tylenol toxicity clinically manifests as AMS with HAGMA. Commonly occurs in pt with
critical illness, poor nutrition, liver disease, CKD, and strict vegetarian diets.
 Use urinary chloride to assess cause of metabolic alkalosis in patients with difficulty assessing volume status. UCl can
be used to asses volume status and saline responsiveness.
 Low (<15) urine chloride suggests reduction in extracellular volume and presence of saline responsive metabolic
alkalosis: post-diuretic usage, vomiting, post-hypercapnic metabolic alkalosis.
 High (>15) urine chloride suggests saline resistance, usually from: active diuretic use, laxative abuse, Gitelman/barter.
 Negative FHX does not exclude ADPKD. 15% of pts have spontaneous mutations. Most common inherited kidney
disorder, incidence 1 of 400-1000 live births (autosomal dominant inheritance). May present with acute pain and
hematuria from bleeding of renal cyst. RCC risk is not higher when compared to the general population.
 Tx of Burkitt Lymphoma: hyper-CVAD (cyclophosphamide, vincristine, doxorubicin, dexamethasone).
 TLS occurs in patients with high tumor burden (high grade lymphoma), or high cell counts (ALL). Hyperkalemia,
hyperphosphatemia, hyperuricemia, hypocalcemia. AKI 2/2 urate and calcium phosphate crystal tubular precipitation.
 Rasburicase rapidly converts uric acid to allantoin, which is 10x more soluble and excreted through kidney. Faster
action than allopurinol, decreasing serum urate levels within 4 hours of administration.
 Allopurinol competitively inhibits xanthine oxidase, blocking metabolism of hypoxanthine and xanthine to uric acid.
Decreases formation of new uric acid, but does not affect circulating uric acid already in the serum.
 Pre-Eclampsia with severe disease: hypertension BP>160/110, plt<100K, Cr >1.1 or double from baseline, AST/ALT
greater than 2x baseline, pulmonary edema, cerebral/visual symptoms.
 Consider steroids if <34 weeks gestation for fetal lung maturation.
 If pre-eclampsia without severe features/disease, and 37 weeks, deliver fetus at that time.
 In secondary FSGS, weight loss in obese patients is associated with improvement in proteinuria in addition to ACEi.
 Cholesterol embolization s/p arterial procedure: AKI, peripheral eosinophilia, hypocomplementemia, eosinophilia on
UA (inflammatory). May lead to digital ischemia/infarction (blue toes) 2/2 digital arterial occlusion, central scotoma
2/2 central retinal artery occlusion, livedo reticularis.
 JNC8: black hypertensive patient: 1st line: CCB (Norvasc/Dilt) or HCTZ. Goal: <140/90. If >60yo: goal is <150/90.
 Blacks have less BP response to anti-RASS Rx than whites. Don’t use ACEi as initial primary sole therapy.
 Try to avoid Norvasc in patients on simvastatin/lovastatin, because they are metabolized via CYP3A4: P450 cystm,
which may increase risk of statin myopathy.
 AASK (African American Study of Kidney Disease and Hypertension) trial: blacks with HTN/CKD did not
demonstrate any difference in more aggressive BP (128/78) vs liberal BP (141/85) goals in slowing GFR decline
 Ethylene glycol/methanol ingestion: IV hydration, fomepizole, urgent HD. Sodium bicarb infusion when pH <7.30
because metabolites (ethylene glycol-glycolate, glyoxylate, oxalate; methanol-formate) penetrates tissues more
effective in the neutral state, which is increased by acidic blood pH. Bicarb is given to ameliorate the toxic metabolic
waste products that cause end organ harm.
 Both ethylene glycol and methanol are completely absorbed from GIT within 1-2 hours of ingestion.
 Treat secondary hyperparathyroidism in CKD with calcitriol. First goal is to normalize Ca and Phos levels, and treat
vitamin D deficiency if present.
 CKD patients with normal calcium/phos levels should be treated with active Vit D analogues to reduce PTH levels
and prevent renal osteodystrophy (BMD). This should be done even if Vit D levels are high/normal because 1a-
hydroxylation of 25-hydroxy Vit D is impaired in CKD patients (PTH will be high). Tx them with oral calcitriol (1,25
dihydroxy vitamin D) or calcitriol analogue (paricalcitol or doxercalciferol) to maintain bone health.
 Calcitriol directly suppresses PTH production by parathyroid glans, protecting bones from osteitis fibrosa cystica.
 Discontinue Vitamin D analogues in setting of hypercalcemia or hyperphosphatemia.
 Current KDIGO (kidney disease improving global outcomes) guidelines do not recommend DEXA in CKD because
DEXA has poor predictive value for distinguishing histologic subtypes of bone disease in CKD patients.
 Primary hyperparathyroidism: high PTH in setting of high Calcium.
 Secondary hyperparathyroidism in CKD: reduced renal production of calcitriol, hyperphosphatemia, hypocalcemia.
 Parathyroidectomy is reserved for secondary hyperparathyroidism refractory to medical therapy (tertiary disease).
 Calculated serum osm: 2(Na) + (BG/18) + (BUN/2.8)
 Pseudohyponatremia 2/2 hyperlipidemia or paraproteinemia (myeloma protein). Hypertriglyceridemia in acute
pancreatitis causes pseudohyponatremia. Measured plasma osm by lab is abnormally normal, because it doesn’t
account for protein/lipids. Calculated plasma osm will be low (which is normal response for this state).
 MCD: MCC idiopathic nephrotic syndrome in kids, 10% of cases in adults. Fusion and dysfunction of epithelial
podocyte foot process of glomerulus, which causes significant proteinuria and other macromolecules in urine.
 MCD presents with acute onset of edema, weight gain, and urine protein-creatinine ratio 5,000-10,000 mg/g. negative
serologic tests, kidney biopsy unrevealing on light microscopy and effacement of podocyte foot processes on electron
microscopy. Tx with steroids to prevent complications of severe nephrotic syndrome (severe symptomatic edema,
thromboembolic events, and infections).
 >80% of patients respond with MCD within 16 weeks of tx. For patients with contraindications to steroids (obesity,
poorly controlled glucose, psychiatric comorbidities), consider calcineurin inhibitors such as cyclosporin.
 Alkylating agents such as cyclophosphamide is reserved for frequently relapsing or steroid dependent MCG patients.
 Acquired hypokalemic periodic paralysis. Associated with thyrotoxic periodic paralysis of hyperthyroidism. A rare
familiar or acquired d/o. Presents with generalized flaccid muscle weakness 2/2 hypokalemia (intracellular K shift)
precipitated by strenuous exercise or a high carb meal. Attacks may also occur spontaneously. Acquired 2/2
thyrotoxicosis usually found in Asian/Mexican men. HPP resolves with tx of hyperthyroidism.
 Barter syndrome: AR inherited renal tubule d/o: metabolic alkalosis, hypoK, low/normal BP, mild volume depletion.
 ACEi/ARB, and direct renin inhibitors are all contraindicated in pregnancy. Since pregnancy causes hypotension from
physiologic changes, don’t need to treat chronic hypertension unless >160/105. Labetalol is relatively safe during
pregnancy, and may help preserve placental blood flow (BB with some a-blocking effects).
 Diabetes insipidus: inability to appropriately concentrate urine in response to increase in plasma osmolality. Central
(inability to secrete ADH) vs nephrogenic (renal resistance to ADH). Serum sodium/osm elevated, urine osm low.
Diagnose first with water restriction test. Not usually with frank hypernatremia, because the hypernatremia stimulates
increased thirst which maintains sodium near upper normal range as long as access to fluids is not impaired. Failure of
urine osm to rise despite rising plasma osm with fluid restriction suggests DI. Tx with DDAVP (desmopressin) for
central DI.
 Cosyntropin stimulation test: Dx adrenal insufficiency: hyponatremia, decreased plasma osm, increased urine osm.
 Hyperkalemia: muscle weakness, cardiac conduction/arrhythmias. Tx: calcium gluconate (stabilizes myocardium by
lowering threshold potential, decrease risk of arrhythmias), insulin/glucose, albuterol (intracellular shift), kayexalate
(works after 2 hours, peaks at 4-6 hours), loop/thiazide diuretics, sodium bicarb (raises pH, shifts K into cells 2/2
buffering process).
 Insulin: drives K into cells by increasing activity of Na-K-ATPase pump in skeletal muscles.
 Acute respiratory alkalosis. Compensation: for each 10 decrease in PCO2, HCO3 falls acutely by 2 due to
intracellular to extracellular shift of H as immediate buffering mechanism. If the respiratory alkalosis is persistent,
renal compensation will occur, leading to reduced PCT reabsorption of HCO3, and after 1-2 days, decrease in total
serum bicarb of 3-4 for every 10 decrease of PCO2.
 Normal AG metabolic acidosis: usually 2/2 CKD (cannot excrete daily fixed acid load), GI loss of HCO3, diversion
of urine through GI conduit system, or retention of H from organic ions excreted in urine as sodium salts. Tx with
sodium bicarb PO tabs to maintain bicarb 23-29 to prevent progression of CKD.
 RTA4: analgesic nephropathy, normal AGMA (nongap acidosis), hyperkalemia.
 Hallmark UA findings of interstitial nephritis: sterile pyuria and leukocyte casts. Inflammatory infiltrate into renal
interstitium that may lead to tubular dysfunction and kidney failure. May be caused by autoimmune disease or drugs
(mesalamine, IBD tx). Mesalamine induced interstitial nephritis may be acute on chronic, even years after exposure,
even in patients who have safely tolerated the medication in the past.
 RPGN: hematuria, RBC casts, variable proteinuria, HTN. Usually with some systemic constitutional symptoms too.
 Increasing dose of BP med is less effective in reducing BP than adding second agent at low dose.
 Lower extremity pitting edema 2/2 amlodipine (CCB ASE) is dose dependent; worse swelling at higher doses
 JNC8: BP<150/90 in patients over >60yo.
 ACCOMPLISH trial (avoiding cardiovascular events in combination therapy in patients living with systolic
hypertension): demonstrated benefit of combo therapy with CCB + ACEi in reducing cardiovascular events compared
with combo therapy with thiazide + ACEi.
 Reduce risk of recurrent stroke/TIA in patients >60 yo with goal SBP<140.
 Masked HTN: normal office BP, but abnormal ambulatory monitoring BP.
 Tx hyperphosphatemia in CKD with phosphate binder: sevelamer or lanthanum or ferric citrate.
 KDIGO guidelines recommend avoid using calcium-containing phosphate binders in patients with vascular disease
(PAD/PVD) 2/2 potential calcium absorption and worsening calcification in the vessel wall. I.E. cinacalcet.
 For CKD patients with anemia, iron deficiency is MCC of hyporesponsiveness to erythropoietin. You must optimize
iron stores first, to maximize response to Epo. Most ESRD pts cannot maintain adequate iron stores via PO. KDIGO
recommends maintaining transferrin sat >30% and ferritin >500 with IV iron. IV iron is more likely to reduce the dose
of ESA (erythropoiesis stimulating agents) compared with PO iron. Target Hgb >11.
 Primary membranous glomerulopathy (MG): idiopathic; antibodies to phospholipase A2 receptors on podocyte
surfaces in 80% of patients. Dx serum anti-PLA2R Ab.
 Secondary membranous glomerulopathy: malignancies (solid organ cancer-lung, colon, breast), autoimmune (SLE,
MCTD), infection (Hep B, C), meds (penicillamine, gold, NSAIDs). Extensive cancer eval is not indicated in MG
patients beyond age-appropriate cancer screening, except for patients with symptoms suggestive of cancer (systemic
constitutional weight loss, hematochezia, etc).
 Protect peripheral veins in renal failure patients for future vascular access (AVF). RIJ>PICC for 6w IV Abx for IE. It
is important to protect peripheral veins in CKD/ESRD patients even if they already have an AVF.
 PICC lines have high risk of causing permanent thrombosis or sclerosis to the veins, that may impede creation of
vascular access for HD. Avoid in CKD/ESRD who are expected to require future HD.
 Subclavian lines have higher risk of subclavian vein stenosis, which may impede return of blood from arm with AVF
HD 2/2 high blood flows. High venous pressure 2/2 subclavian stenosis can lead to arm edema and AVF failure.
 Inserting needles into recently created AVF will damage the walls of the vein (haven’t had long enough time to
arterialize, which requires 2-3 months). Don’t inflate BP cuff on that arm to avoid injury/thrombosis if fresh AVF.
 Pituitary apoplexy: acute hemorrhage into pituitary gland. May cause visual field deficits 2/2 ocular nerve
compression, and hypopituitarism. ACTH deficiency (cortisol deficiency, hypotension).

Cardiology:
 Tetralogy of Fallot: VSD, RVH, PS, over-riding aorta. Most common structural disorder after repair is PR.
 When to order TTE: asymptomatic with 3/6 systolic, late/holosystolic/diastolic/continuous murmur.
 After AF ablation, anticoagulation is based on CHADSVASC2 risk stratification rather than rhythm status.
 Atrial fibrillation Clopidogrel Trial with Irbesartan for prevention of Vascular Events (ACTIVE-W) trial compared
warfarin vs with aspirin/clopidogrel and found that aspirin/clopidogrel was inferior to warfarin for stroke prevention
with no statistically significant difference in bleeding.
 Avoid exacerbating LVOT obstruction in HOCM via: preload (diuretics, squat-to-stand), afterload (expiration,
vasodilator therapy), or contractility (digoxin). Tx HOCM patients with BB, CCB, or disopyramide.
 Who should get high intensity statin: LDL>190, 40-75 yo DM with ASCVD risk >7.5%, patients with known
atherosclerotic disease (CAD, CVA, PAD, etc).
 High intensity statin: atorva/Lipitor 40-80, rosuva/Crestor 20-40, simva/Zocor 80
 Low intensity statin: fluva/lescol 40, lova/mevacor 20, prava/Pravachol 10, simva/Zocor 10.
 Duke exercise stress: >+6 (low risk), -10 to +4 (intermediate risk), <-11 (high risk).
 Bicuspid AV is 2nd MCC of AS (1st is calcific tricuspid AV), and 2nd MCC of AR (1st is Aortic root dilatation).
 Bicuspid AV is more prevalent among patients with turner’s, coarctation, and interrupted aortic arch.
 HFpEF patients are very volume sensitive. Important to maintain euvolemia with diuretics. No meds for HFpEF have
shown to decrease mortality.
 Treat low-pressure cardiac tamponade with IVF hydration and pericardiocentesis. Causes of LPCT: malignancy, TB,
chronic disease that causes concomitant dehydration with pericardial effusions.
 Evidence of tamponade: diastolic inversion of RH chambers, ventricular septal shift/bowing, IVC plethora, JVD,
pulsus paradoxus. (JVD and pulsus paradoxus absent in low-pressure tamponade).
 INTERHEART trial: modifiable risk factors in descending order: HLD, smoking, stress, DM, HTN, BMI, EtOH, lazy,
bad diet. Prevalence 90% in patients with acute MI (recognized risk factors in these patients).
 FREEDOM trial: CABG vs PCI in diabetics with multi-vessel CAD; lower primary composite endpoint of death, MI,
CVA. Statistically significant reduction in occurrence of death and MI in CABG patients. CVA rates were higher in
CABG group.
 When to do CABG: symptomatic despite optimal medical therapy, specific angiographic findings (left main disease,
multi-vessel disease with proximal LAD involvement), reduced LV function, or DM.
 Surgical Treatment of Ischemic Heart Failure (STITCH) trial: no relationship between results of viability imaging and
effectiveness of CABG.
 What is viability imaging? Test with radionuclide tracer that is taken up by viable myocardial tissue. May demonstrate
hypoperfused regions that may have functional improvement s/p revascularization. However, if the entire heart has
poor uptake, it may look normal.
 Avoid vasodilators (dipyridamole, adenosine, and regadenoson) that may cause bronchospasms in COPD patients for
nuclear stress test. Consider using dobutamine stress test instead if active wheezing.
 Follow up severe asymptomatic AS with TTE q6-12 months as outpatient. TAVR not approved for bicuspid AV.
Only consider TAVR if there are significant co-morbidities in patients that are not good operative candidates.
 Takotsubo: no occlusion on cath, hypokinesis of the mid/apical LV on ventriculography. Tx with BB and ACEi.
 PDA/ASD/VSD can predispose to Eisenmenger syndrome, which causes RL shunting, PAH, clubbing/cyanosis.
 Acute low output decompensated CHF XC: treat with inotropic medications such as dobutamine/milrinone. Milrinone
worsens hypotension, and is cleared by kidneys, so in AKI, give dobutamine to augment contractility and cardiac
output.
 IABP: inflates during diastole, augments coronary/systemic perfusion. Deflates during systole, LV afterload.
 When to give Abx PPX prior to dental procedure: hx of previous IE, hx of heart transplantation, prosthetic valve, and
some complex congenital heart disease.
 Treat bare metal stent with >1 month of ASA/Plavix. If AF develops during that time, CHADSVASC2 for warfarin
(triple therapy). DES requires longer duration of dual therapy depending on the type of stent. Treatment with
ASA/coumadin does not optimally prevent acute stent occlusion in patients s/p PCI.
 Randomized Evaluation of Long Term Anticoagulant Therapy (RE-LY) trial: numeric excess of MI observed with
dabigatran/pradaxa.
 Adults with ostium secundum ASD often present with atrial arrhythmias. Fixed S2 split, RV heave (parasternal
impulse at LLSB), midsystolic murmur at 2LICS (pulmonic flow murmur, tricuspid diastolic flow rumble). EKG
shows RBBB and RAD.
 Ostium primum ASD EKG findings show 1st degree AV block, LAD, RBBB.
 Planned pregnancy is a class I indication for MV repair in patients with severe MS even if asymptomatic. If young,
consider candidacy for MV balloon valvuloplasty.
 Surgically repair AAA if >5.5 cm in men and >5.0 cm in women. (EVAR vs open approach). Annual surveillance
peripherally if <5cm.
 Consider catheter ablation therapy for patients with frequent symptomatic PVCs with decline in LV function despite
treatment with β-blocker therapy. PVCs are common and usually benign. But symptomatic or frequent PVCs
(>10,000 PVCs/24 hours or >10% of all beats) require treatment. 30% of patients with frequent PVCs develop PVC-
induced cardiomyopathy and progressive LV dysfunction. First-line therapy for symptomatic or frequent PVCs is β-
blocker or CCB.
 Who is eligible for biventricular pacemaker? Must meet all these indications: on guideline-directed medical therapy, a
reduced EF ≤35%, a wide QRS ≥150 ms or LBBB, and NYHA functional class III or IV symptoms.
 HTN <20w pregnancy: chronic. Treat if >150 with labetalol or methyldopa.
 Valsalva decreases preload, which worsens HOCM murmur.
 Hold BB and nondihydropyridine CCB 48 hours prior to stress test. Okay to continue dihydropyridine CCB.
 Women’s health initiative study: 100mg/d ASA decreased risk of CVA, MI, cardiovascular death in women >65.

Cardio Q#35: TBC.

Restarted at Q 47:
 For patients with new onset HF and e/o CAD, myocardial perfusion stress test is indicated. Treatable causes of new
CHF include: CAD, valvular disease, thyroid dysfunction, and alcoholism.
 Cardiac MRI may be indicated for infiltrative/inflammatory disease such as amyloidosis, Wilsons, hemochromatosis,
sarcoidosis, myocarditis.
 Perform LHC on a diabetic with constant stable angina refractory to medical management to evaluate for
revascularization therapy with PCI.
 An ischemic perfusion stress test alone is not an indication for LHC if the patient is asymptomatic.
 TEE approximates the LA and MV for better visualization.
 ABI <0.90 is diagnostic of PAD. Exercise and cilostazol are mainstay for stable symptomatic PAD. Give antiplatelet
tx if symptomatic PAD, hx revascularization, or amputation 2/2 PAD.
 PLATO trial (platelet inhibition and patient outcomes trial): use of ticagrelor was associated with 2% absolute risk
reduction in occurrence of CV death, MI, and CVA when compared to clopidogrel (Plavix).
 P2Y12 inhibitor (clopidogrel/plavix, prasugrel/effient, ticagrelor/brilinta) should be continued minimum 1 year s/p
PCI.
 EPHESUS (eplerenone post-MI HF efficacy and survival) trial: 2007 ACC/AHA guidelines recommends use of
aldosterone antagonist (eplerenone, Aldactone) to all patients s/p NSTEMI who are on ACEi, have LVEF<40%, or
have either CHF symptoms, or DM.
 Cardioselective BB for HFrEF: metop succinate, carvedilol, and bisoprolol. Increase dose slowly every 1-2 weeks
until max effect achieved. Higher doses of BB have improved outcome on mortality, cardiac function/EF, and reduced
symptoms.
 Metop succinate: long acting; metop tartrate: short acting.
 Platypnea/orthodeoxia: SOB when upright, improves with recumbence. Hypoxemia improved with recumbence.
Think of RL shunting in ASD or PFO; when recumbent, there is deformation of atrial septum and increased RA
pressure, which causes RL shunting, causing cyanosis (blood bypasses pulmonary circulation).
 Inferior MIRV dysfunctionannular dilatationTV regurgitationforamen ovale stretchesPFOshunting.
 VSD: acute dyspnea, pulmonary edema, holosystolic murmur @ L sternal border that does not ∆ with inspiration.
 CRT: cardiac resynchronization therapy with biventricular pacemakers: used for patients with EF<35%, NYHA class
3-4 with symptomatic CHF, and LBBB or QRS >150.
 AICD: defibrillator and pacemaker: EF<35%, NYHA class 2-3 symptoms.
 LVAD: LV assist device: for refractory heart failure symptoms despite optimal medical therapy; bridge to cardiac
transplantation.
 ETT is for intermediate risk patients without EKG changes (LBBB, ST changes, LVH, WPW, paced rhythm).
 ETT has more information than stress echo about patient prognosis and functionality.
 Echo stress is better for visualization in localizing the severity, site, and extent of the ischemia in myocardium.
 LBBB in ETT: exercise may cause abnormal septal motion 2/2 conduction delay, with false positive septal leads.
 CMR: cardiac MRI: myocardia/pericardial disease, and aortic pathology. Evaluates extent of fibrosis, and may help
with determining the extent of MI and potential viability of tissue.
Restart at Q59.

Severity of aortic stenosis


Severity of mitral stenosis Mean gradient Aortic valve area
Degree
Degree of mitral stenosis Mean gradient Mitral valve area (mmHg) (cm2)
Mild mitral stenosis <5 mmHg >1.5 cm2 Mild <25 >1.5
Moderate mitral stenosis 5 - 10 mmHg 1.0 - 1.5 cm2 Moderate 25 - 40 1.0 - 1.5
Severe mitral stenosis > 10 mmHg < 1.0 cm2 Severe >40 < 1.0
Very severe >70 < 0.6
CARDIOLOGY
 ToF: large subaortic VSD, infundibular/valvular PS, aortic override, RVH. PR is MC structural d/o that occurs
following ToF repair. TR findings: RH overload, parasternal (RV) lift, and soft systolic pulmonary outflow murmur.
Single S2 on auscultation because function of pulmonic valve is sacrificed during repair.
 PR diastolic murmur increases during inspiration; 2LICS.
 AR diastolic murmur decreases with inspiration.
 TR holosystolic murmur at LLSB that increases with inspiration.
 Tx hemodynamically stable narrow complex tachycardia (SVT) with valsalva or adenosine. Presents with neck
pulsations (simultaneous contraction of atria and ventricles).
 AVNRT: atrioventricular nodal reciprocating tachycardia: regular rhythm w/ no obvious visible p waves. Comprises
of 66% of SVT.
 Adenosine is highly effective at termination of nodal-dependent rhythms, and can help identify the underlying
etiology. For example, continued atrial activity (P waves) during AV block can help identify atrial flutter and atrial
tachycardia. Adenosine may cause nausea, flushing, chest pain, and sense of dread. Patients with bronchospastic lung
diseases should not receive adenosine. Give 6mg, then 12mg if does not break rhythm.
 Amiodarone toxicity: thyroid, liver, pulmonary, neurologic toxicity.
 Ibutilide: IV Vaughan-Williams class 3 antiarrhythmic drug for pharmacologic cardioversion of A fib.
 ACC/AHA guidelines recomend that all patients with ACS (UA, NSTEMI, STEMI) treated medically or with stent
(BMS, DES) should be given P2Y12 inhibitor therapy (clopidogrel/Plavix, prasugrel/effient, ticagrelor/brilinta) in
addition to ASA for at least 12 months
 Patients who have stents in absence of ACS (stable angina) also need DAPT until endothelialization of the stent is
completed, for risk of acute stent thrombosis. BMS: 1 month. DES: 1 year.
 Two MCC of dyspnea in post-cardiac transplant patients are rejection and cardiac allograft vasculopathy. Prevalence
of cardiac allograft vasculopathy is 50% by year 5 post transplant and is MCC mortality in patients after year 1 of
transplant (MCC reduced LVEF too). Because the transplanted heart is denervated at the time of transplant,
vasculopathy and ischemic may occur without classic symptoms of angina (chest pain is absent). Diagnose via
coronary angiography; may consider dobutamine stress echo for lower risk patient (if negative coronary angio
recently). Incidence of organ rejection after 1 year is low, unless they are noncompliant with their meds.
 Denervated transplant hearts (vagus nerve) do not have parasympathetic innervation on the SA node to slow down the
heart. Normal heart rate in transplant patients is 90-110 bpm. Tachy is not unusual.
 Patients with AICD and planned surgery, the ICD should be reprogrammed immediately before the procedure to
asynchronous pacing mode (DOO) with disabling of tachycardia detection and shocking function (both turned off).
 DOO mode: asynchronous mode: allows continued pacing of atrium and ventricle but without the device sensing the
cardiac response, thereby avoiding suppression of pacing due to electrical interference that the device might interpret
as an elevated heart rate (that is, oversensing).
 Disabling the shock function will eliminate false detection of tachyarrhythmia 2/2 electrical interference.
 Applying external magnetic field to a cardiac device will change different functions. Magnet to pacemaker: induces
asynchronous pacing (pacing regardless of what is sensed). Magnet to defibrillator: disables shocking function
without changing pacing programming.
 Evaluation of patients undergoing surgery with implanted cardiac electronic device: what type of device? Is the
patient pacemaker dependent? Will surgery be performed with instruments that result in electromagnetic interference
in the vicinity of the device or its leads?
 Ostium primum ASD: dyspnea, previous AFib, volume overload of RH, elevated CVP, RV lift/heave. Systolic
murmur at base 2/2 increased flow across RVOT from the L to R shunting. Fixed S2 split=ASD. Apical systolic
murmur 2/2 MR from mitral valve cleft. EKG: left axis deviation, 1st degree AV block, interventricular conduction
delay.
 Ostium secundum ASD or coronary sinus ASD or sinus venosus ASD: RH volume overload, but absence of MV
disease. No murmur of MR. EKG may be normal or show 1st degree AV block and incomplete RBBB.
 PFO: normal echo, normal physical. Asymptomatic.
 VSD and cardiogenic shock s/p 3-7 days after MI: warrants emergency surgery (CABG and VSD patch repair).
Suspect if new holosystolic murmur 3-7 days s/p MI. high mortality; infarcted myocardium is friable and makes
surgical closure of the septal defect difficult.
 Acute ventricular free wall rupture also occurs 3-7 days post MI with high mortality rate. Presents with pericardial
tamponade 2/2 hemopericardium, pulseless electrical activity, and death. Emergent pericardiocentesis and subsequent
surgical reconstruction can improve survival.
 Black patients with NYHA class 3-4 failure should be on hydral/Isordil in addition to standard heart failure medical
regimen to improve symptoms and reduce mortality. SE: peripheral edema, headaches.
 Optimal therapy for heart failure: ACEi, BB, aldosterone antagonist. ARB (losartan) is not recommended because of
hyperkalemia and AKI.
 Cardiac resynchronization therapy (CRT) may be considered if patients have HF + long QRS indicating
dyssynchrony. Such as: LVEF<35% in sinus, with LBBB, and with QRS>150ms with moderate-severe symptoms
NYHA class 3-4 despite optimal medical therapy.
 Staph aureus IE: eval with TEE if patient has heart block (any conduction abnormality) to r/o possible perivalvular
abscess. Abscesses present up to 30-40% in IE and risk is higher if bicuspid AV. Cardiac CT not extensively studied
for diagnosing myocardial infection. Cardiac MR (CMR) is effective, and can be used if TEE equivocal but still
strong clinical suspicion.
 Hallmark of UE PAD is a difference in SBP between two arms (>15mmHg). Impaired arterial flow with exertion
causes claudication in one limb, with associated neurologic symptoms suggesting subclavian steal syndrome.
 Subclavian steal: stenosis in L subclavian artery proximal to take-off of the vertebral artery; causes retrograde blood
flow from CNS to arm, resulting in neurologic symptoms (dizziness, fatigue). UE PAD is marker for significant
ASCVD; primary treatment is aggressive therapy including antiplatelets. Tx with angioplasty or stenting with surgical
bypass if symptomatic.
 Pulsus paradoxus: drop in SBP >10mmHg during inspiration with severe pericardial disease (tamponade, constrictive
pericarditis), asthma, and COPD.
 Trifascicular block (not true): first degree AV block, bifascicular block. True trifascicular block would have complete
AV block. No indication for pacing first degree AV block and bifascicular block if asymptomatic from bradycardia.
Risk of progression to complete hear block is 2-3% annually.
 Indications for permanent pacing: high grade AV conduction abnormalities with likelihood of progression to complete
heart block or life threatening sudden asystolic, 2nd degree mobitz type 2 block, alternating BBB, AF with pauses
greater than 5 seconds, and complete heart block.
 ACEi decrease both cardiovascular and all cause mortality in patients with chronic IHD, especially in those with prior
MI, LV systolic dysfunction or CHF. AHA/ACC recommends treatment goal of <140/90 for CAD.
 JNC8: SBP<140/90 for all patients <60yo.
 GISSI: Gruppo Italiano per lo studio della sopravvivenza nell’infarto miocardico: prevention study: patients with prior
MI who take fish oil have 20% reduction in mortality rate. Not adopted by AHA/ACC. Fish oil helps lower TG levels.
 Pts with NSTEMI/UA, TIMI risk score is used to determine whether conservative or early invasive strategy is
warranted. PCI vs CABG (CABG if multivessel disease with reduced LVEF). Revascularization therapy if Left main
stenosis, depressed LVEF and ACS.
 CABG should be delayed for 5 days to allow discontinuation and excretion of oral P2Y12 inhibitor (Plavix).
 Surgical AV replacement for symptomatic severe AS. Low mortality rates for patients younger than 70yo (1-3%).
 AV repair is only restricted to AR and anatomically favorable AV and root anatomy. Not for severe calcifications.
 Balloon valvuloplasty: pediatric patients with severe AS. Used for high risk patients with severe COPD or calcific
aortic stenosis with hemodynamic instability or decompensation. Severe AR is contraindication for valvuloplasty.
 TAVR for severe symptomatic AS with high or prohibitive surgical risk. Assess risk via STS score (society of
thoracic surgery adult cardiac risk score). >4% score is candidacy for TAVR. If markedly calcific AV, surgical
replacement is superior to TAVR.
 Left atrial myxoma: most common benign cardiac tumor. Mobile with pedunculated stalk. May cause consittuioonal
symptoms (fatigue, dyspnea, fever, weight loss) related to tumor cytokine production. May also cause neurologic
symptoms or systemic sequelae from systemic embolization of the tumor fragment or associated thrombi. Mass may
involve the MV and cause tumor plop that mimics MS.
 Carney complex: AD disorder associated with pigmentation abnormalities (blue nevi), schwannomas, endocrine
tumors, and atrial myxoma.
 Papillary fibroelastoma are also mobile with pedunculated stalk, but are usually on L cardiac valves and LVOT.
 Vast majority of acute pericarditis can be managed as outpatient. Only high risk features of acute pericarditis needs
inpatient monitoring including: fever, WBC, acute trauma, elevated cardiac enzymes, immunocompromised state, oral
anticoagulation usage, large effusion, or evidence of cardiac tamponade. Treat with anti-inflammatory agents such as
NSAID, ASA, indomethacin and steroids if refractory. Steroids may increase risk of recurrent pericarditis.
Pericardiocentesis is indicated only if tamponade or for diagnostic management. During inspiration, negative thoracic
pressure causes increased pressure into RV that causes the interventricular septum to bulge towards the LV, leading to
decreased filling of LV. At the same time, RV volume is markedly diminished and sometimes can collapse.
 Beck’s triad of tamponade: JVD, hypotension with narrow pulse pressure, and muffled heart sounds.
 WPW (Wolff Parkinson White syndrome): Preexcitation on EKG with hx of palpitations and syncope. Undergo
electrophysiology study. EKG shows delta waves (slurring of QRS complex) from early ventricular depolarization
from conduction through accessory bypass tract pathway. Episodes of syncope are 2/2 SVT (orthodromic or
antidromic reciprocating tachycardia) or preexcited Afib. Electrophysiology procedure is not only diagnostic but also
therapeutic as it can ablate the accessory pathway. Antiarrhythmic agents are second line if patient has an abnormal
pathway close to the AV node that is not amenable to ablation therapy. BB/CCB/AV nodal blockers are unsafe if
patient has anterograde conduction down the accessory pathway during AFib, because they may block AV node and
promote 1:1 rapid conduction from atrium to ventricular, and induce VFib arrest.
 Workup reversible causes for new onset CHF: TSH, viral, rheumatologic disease, giant cell myocarditis (biopsy).
 Hyperthyroidism: tachycardia, hyperdynamic precordium, palpitations, weight loss, loose stools.
 Hyperthyroidism causes sympathetic overstimulation, tachy, increased myocardial contractility, SVR is decreased and
causes wide pulse pressure.
 Marfan syndrome: annual imaging surveillance of the aorta (widened aortic root; aneurysm). High risk of acute aortic
syndrome (dissection +/- rupture). Severity of disease is graded by width, length, and location of aorta involved.
 ACC/AHA: recommends 6 month f/u TTE after diagnosis, with annual surveillance if root <4.5cm in diameter.
 If aortic root diameter is >4.5cm, then more frequent imaging should be considered.
 +Exercise stress EKG test indicates CAD, and warrants coronary angiography/catheterization. Exercise stress EKG
test not only diagnoses CAD, but evaluates adequacy of medical therapy and evaluates functional status. Positive test
is diagnostic of CAD (>70% obstruction) if >1mm ST depression with exercise in >2 contiguous leads.
 Duke treadmill score (DTS): exercise time (minutes) – (5 x ST segment depression) – (4 x angina score).
 Angina score: 0=asymptomatic; 1=nonlimiting angina; 2=exercise limiting angina). Scores <-11 are high risk, and
scores >5 are low risk. High risk scores correlate with left main, or proximal left anterior descending (LAD) artery
disease.
 Stress echo or myocardial perfusion imaging studies can help localize ischemia to a vascular territory.

INFECTIOUS DISEASES.
 For TB meningitis, give Decadron 12mg/day for 3 weeks, followed by 3 week taper. Mortality benefit. Tx for 9-12
months instead of regular 6-9 months.
 CNS toxo in AIDS patients usually is 2/2 reactivation rather than primary infection. Treat empirically, and monitor
clinical/labs for 1-2 week for improvement.
 Primary CNS lymphoma in AIDS: CD4<50, with single mass in brain; strongly associated with EBV infection.
 Serum and CSF CrAg are very good tests; negative essentially rules it out and argues against Cryptococcus.
 PML: progressive multifocal leukoencephalopathy: white matter non-inflammatory changes with minimal
enhancement found on MRI.
 Patients with VAP should respond to Abx therapy within 72 hours.
 Adding rifampin to vanco for MRSA bacteremia hasn’t shown to improve clinical outcomes.
 If MIC for vanco>2, then switch to daptomycin. Target trough 15-20 for bacteremia and severe infection.
 Prior to starting RIPE: baseline LFT, P7, plt count.
 Ethambutol SE: retrobulbar neuritis, with decreased green/red color discrimination or decreased visual acuity. Also
watch out for rash and peripheral neuritis.
 Isoniazid SE: hepatotoxicity, peripheral neuropathy, rash, drug induced lupus.
 Pyrazinamide SE: hepatotoxicity, rash, GI upset, hyperuricemia.
 Rifampin SE: hepatotoxicity, rash, red/orange secretions, GI upset.
 Streptomycin SE: ototoxicity (hearing and vestibular dysfunction: audiogram and Romberg respectively),
neurotoxicity, and nephrotoxicity. Similar to other aminoglycosides.
 Rocky mountain spotted fever (RMSF): fever, lymphocytic meningitis, headache, petechial rash, tick exposure. Up to
20% with meningoencephalitis. Maculopapular rash around ankles/wrists, spreads to trunk, sparing face.
Characteristic involvement of palms/soles. Thrombocytopenia, transaminitis. Tx with Doxy, rapid clinical response
and improvement to therapy. During acute illness, Rickettsia rickettsii antibodies undetectable because of insufficient
time to mount serologic response to infection. Test a convalescent serum specimen after 2 week after symptom onset
for retrospective confirmation of RMSF infection. Skin biopsy may reveal vasculitis, confirmed by
immunohistochemistry.
 Enteroviral rash is morbiliform characteristic
 Heartland virus: from same tick that causes ehrlichiosis.
 Hospital acquired (>3 days of inpatient) bacterial gastroenteritis (other than CDI) is <1% of stool cultures. Rare.
 Cocci meningitis: CSF with lymphocytic pleocytosis, high protein, low glucose, eosinophil elevation up to 70%,
complement fixing antibodies (IgG) more sensitive than culture in diagnosis, present up to 90% of patients. Tx with
fluconazole (intrathecal ambisome only if nonresponsive).
 For unknown syphilis duration (+serum RPR with +fluorescent treponemal antibody absorption testing), with
cognitive symptoms, should have CSF sent for protein, glucose, cell count+diff, VDRL testing. If positive for
neurosyphilis, then treat with IV PCN, but if negative, treat with IM PCN for late latent syphilis.
 For TB patients, start with 2 month initial treatment of RIPE, and finish off with 4 month continuation therapy of
Rif/INH. But in these patients, you need prolonged 7 month continuation therapy: patients who do not receive
pyrazinamide during initial phase, patients receiving qweekly INH and rifapentine whose sputum cultures still
positive at end of initial treatment phase, and in patients with cavitary pulmonary TB whose sputum cultures still
positive at end of initial treatment.
 In absence of drug resistance, 6-9 month of total TB treatment is sufficient; if resistance, then >9 month.
 Pyrazinamide is contraindicated in patients with active gout because it inhibits renal tubular excretion of uric acid.
 All adults should receive Tdap vaccine; childhood version (DTaP) has waning immunogenicity into adult years; will
need a single booster with Tdap regardless of timing of previous DTaP booster.
 Positive PPD: >5mm if recent contact with active TB. >10mm in patients with homeless vietnamese IVDU.
 Tx LTBI with 9 months of INH or 12 weeks of DOT weekly INH/Rifapentine.
 Tx Aeromonas hydrophila associated necrotizing skin infection with doxy+Cipro or doxy+CTX. Aeromonas is found
in freshwater environments or brackish water.
 Treat strep pyogenes (group A beta hemolytic strep) with PCN + Clinda if toxic shock/nec fasc.
 Patients with pyelo who remain febrile >72 hours need imaging (CT or US) to rule out abscess. Patients who are
predisposed to abscess include poorly controlled diabetics and nephrolithiasis.
 Occupational or non-occupational HIV exposure warrants triple therapy with tenofovir-emtricitabine and raltegravir
while awaiting results of HIV testing.
 Standardized encephalitis evaluation includes LP, MRI, and EEG. (obtunded, fevers, and high WBC on CSF).
 Nonconvulsive status epilepticus may be focal or generalized and has been reported with viral and autoimmune
encephalitides. It is associated with a delay in initiating antiepileptic therapy and an increased risk for death.
 Patients with late (terminal) complement deficiencies (C5-C9) may present with recurrent, invasive meningococcal or
gonococcal infections. C5 deficiency confers impaired chemotaxis and absent serum bactericidal activity. C6, C7, and
C8 deficiencies result in absent serum bactericidal activity, and C9 deficiency results in impaired serum bactericidal
activity.
 Brain abscess 2/2 odontogenic source needs IV PCN and flagyl to cover Fusobacterium, Prevotella, bacteroides, and
strep. Do not need pseudomonal coverage unless recent brain surgery.

Common variable immunodeficiency (CVID) involves B- and T-cell abnormalities, and the usual manifestation is
hypogammaglobulinemia. Bacterial infections, often of the sinus tract and lungs, are common, and the immune
dysregulation seen in CVID is associated with autoimmunity and malignant disease. However, CVID is not associated
with recurrent meningococcal infection.

Selective IgA deficiency is a common B-cell immunodeficiency. Most patients are asymptomatic or have sinopulmonary
infections or GI involvement with IBD, sprue-like illness, or celiac disease. Giardiasis may also be seen.
Classical complement pathway (C1, C4, C2) deficiencies are often associated with a rheumatologic dz, such as SLE,
vasculitis, dermatomyositis, or scleroderma. Frequency of infection is relatively low in those with C1, C4, or C2
deficiency compared with deficiencies of other complement components. When they do occur, common infections are
caused by encapsulated bacteria, especially strep pneumo.

Cytomegalovirus is a common complication of transplantation, especially in the first few months after transplantation
when immunosuppression is typically highest, and patients who have just finished prophylaxis against cytomegalovirus
are at risk for reactivation. The colon and esophagus are common sites for CMV disease after transplantation.

Polyoma BK virus: tubulointerstitial nephropathy in s/p kidney transplant patients.

You might also like